Predictions Only

You might also like

Download as pdf or txt
Download as pdf or txt
You are on page 1of 89

PHILIPPINE NURSING LICENSURE

EXAMINATION (PNLE)

REVIEWER
COMPILATION OF BOARD EXAM
NOVEMBER 17 & 18, 2018

SUBJECTS COVERED

NURSING PRACTICE I: Community Health Nursing

NURSING PRACTICE II: Care of Healthy/At Risk Mother and Child

NURSING PRACTICE III: Care of Clients with Physiologic and Psychosocial Alterations, Part A

NURSING PRACTICE IV: Care of Clients with Physiologic and Psychosocial Alterations, Part B

NURSING PRACTICE V: Care of Clients with Physiologic and Psychosocial Alterations, Part C

COMPILED BY: TITO A.RN


NURSING PRACTICE I: Community Health Nursing
November 17 & 18, 2018
Situation - According to the World Health Organization (WHO), there are major causes of
non-communicable disease that pose challenges to the live and health of millions of
people and threaten economic and social development of countries.

1. Which of the following are the four chronic disease referred to by WHO?

1. Cardiovascular Disease
2. Cancer
3. Dementia
4. Arthritis
5. Diabetes Mellitus
6. Chronic Obstructive Lung Disease
A. 1,2,3 and 4
B. 1,2,5 and 6
C. 2,3,4 and 5
D. 2,4,5 and 6

2. The WHO reveals that the NUMBER ONE cause of death from chronic diseases worldwide, using the
2015 estimate, is _______.
A. Tuberculosis
B. Diabetes Mellitus
C. Ischemic Heart Disease
D. Pneumonia

3. In the Philippines, which is the TOP Killer according to the Department of Health (2009 data)?
A. Asthma
B. Cancer
C. Tuberculosis
D. Cardiovascular Diseases

4. When studying chronic diseases, the multifactorial etiology of illness is considered. What does this
imply?
A. Single organism that causes the disease, such as cholera, must be studied in more detail.
B. Focus should be on the factors or combinations and levels of factors contributing to disease.
C. The rise in infectious and communicable disease must be the main focus.
D. Genetics and molecular structure of disease is paramount.

5. Determinants of health to address the development of cancer in a community include:

1. Proximity of the community to chemical plants that emit poisonous gases.


2. High percentage of tobacco use among the residents.
3. Prevailing diet high in processed food and fat.
4. Availability of the health facilities.
5. Men age of women population.
A. 3,4 and 5
B. 1,2 and 3
NURSING PRACTICE I: Community Health Nursing
November 17 & 18, 2018
C. 1,3 and 4
D. 2,4 and 5

Situation - As a nurse, knowledge of the legal aspects in nursing is essential to safeguard self and clients
from legal complications. Clients are becoming increasingly aware of their health rights, thus you should
recognize your professional boundaries and consequences of nonconformity.

6. Your co-worker got a thank you card from a patient’s family with an accompanying photo of her with the
family. She asked you if it would be alright to post the post on facebook. Which would be your BEST
response?
A. “I think that would be okay, but you should ask permission from our head.”
B. “Yes, as long as you ask the family if that would be okay.”
C. “No, that could lead to a malpractice suit by the patient or family.”
D. “No, posting pictures of patients and families on social media is not acceptable.”

7. Identify which action is acceptable as an exception to a nurse’s obligation regarding confidentiality:


A. Leaving printouts of lab reports on the desk in the physicians’ lounge
B. Discussing a patient’s care with someone who is not involved in patient care
C. Discussing a patient’s condition in a public place as long as his name is not mentioned
D. Reporting certain disease to public health authority

8. The central question in any charge of malpractice is whether the prevailing standard of care was met.
Which of the following statements are TRUE about standards of nursing care?

1. Basic prudent nursing care is a standard.


2. Any health care provider can determine standards of care.
3. Standards of care are never changing.
4. Standards are based on the ethical principle of non- maleficence.
5. National standards of nursing practice are standards for all nurses.
A. 1,3,4
B. 2,3,5
C. 1,4,5
D. 2,4,5

9. Select the nursing responsibility which can NEVER be delegated:


A. Rotation schedule
B. Evaluation
C. Accountability
D. Complex Procedure

10. For safe nursing practice, you should observe some of the do’s and do not in performing your
professional duty. Which ones are NOT acceptable standard of actions by the nurse?

1. Keeping current year license to practice


2. Being a witness to a patient’s will
3. Following policies and procedures
NURSING PRACTICE I: Community Health Nursing
November 17 & 18, 2018
4. Protecting patients from injuring themselves
5. Accepting money or gift from patients
6. Giving practical advice contrary to the doctor’s orders
A. 1,4, 6
B. 1,3,4
C. 2,4,5
D. 5,6

Situation - Nurse Betty knows that ethics is important in community health nursing practice and that ethical
decisions are conducted in a universal and standard framework.

11. One step in ethical decision- making is to place an ethical issue or dilemma within a meaningful context.
The rationale for this step is:
A. The nature of ethical issues and dilemmas determines the specific ethical approach used.
B. People cannot make sound ethical decision if they cannot identify ethical issues and dilemmas
C. Multiple factors affect the interpretation and justification of the ethical issues and dilemmas
D. Health professionals cannot avoid choice and action in applying ethics in their work setting.

12. The historical character in nursing whose work in environmental health, providing care to destitute
people, and disease prevention establishing the nurse as a leader and community-oriented was _______.
A. Lilian Wald
B. Virginia Henderson
C. Margaret Sanger
D. Florence Nightingale

13. Ethical decision making is the component of ethics that focuses on ____.
A. The result of the decision
B. The process of how decision are made
C. The factors impinging on making the decision
D. Who makes the decision

14. The ethical principle that empowers the client (groups or communities) to make knowledgeable
decision is _.
A. Act in accordance to client’s wishes
B. Act in client’s best interest
C. Offer franks and independent advice
D. Keep the client properly informed

15. When nurse Betty cannot meet a need of the client, it is most appropriate for her to:
A. Refer the client to an agency that may be able to give assistance
B. Suggest that the client discuss the need with the physician.
C. Encourage the client to pay someone to meet that need.
D. Inform the client that insurance will not pay for the care.

Situation - Nurse Rosa’s makes home visits to assigned barangays and meets families of different types
NURSING PRACTICE I: Community Health Nursing
November 17 & 18, 2018
16. One such family is a married couple who has two biologic children living with them as well as a child
from the wife’s first marriage. What type of family is evident?
A. Homogenous
B. Extended
C. Blended
D. Nuclear

17. Nurse Rosa’s encounters a married couple who is raising three children. Recently, the wife’s mother
moved in. This family should be assessed as ___.
A. Nuclear
B. Extended
C. Alternative
D. Blended

18. When Nurse Rosa’s assesses a family, which family task would she consider having HIGHEST PRIORITY
for health family functioning?
A. Reproduction of new family members
B. Physiologic maintenance and safety
C. Allocation of family resources
D. Maintenance of order and authority

19. Which documentation of family assessment indicates a healthy and functional family?
A. Member provide loving and mutual support
B. Under stress, members turn inward so as not to be burden others.
C. Members believe they can depend in each other.
D. Husband holds dominant power over his wife

20. Which of the following scenario BEST demonstrates a continuing healthy family?
A. A couple renews their marital relationship after their children become adults
B. A couple requires their adolescent children to attend church services three times a week
C. One parent takes care of children. The other parent earns income and maintains the home
D. A family has strict boundaries that require members to address problems within the family.

Situation - Interdisciplinary teamwork is an important model for delivering health care to patients.
Integrating services among many health providers leads to better outcomes delivered to under served
populations and communities with limited access to health care.

21. Professional group productivity and member satisfaction are best achieved by which of the following
leadership styles?
A. Democratic
B. Authoritarian
C. Laissez-faire
D. Autocratic

22. As a member of a professional work-related group you know that:


A. Functional goals are assigned by management
NURSING PRACTICE I: Community Health Nursing
November 17 & 18, 2018
B. The leader is responsible for overall functioning
C. The members summarize progress and allocate assignments
D. The cooperation of all members is required.

23. The nurse is making a referral for the client. What should be the first consideration for the nurse to do?
A. Acceptability of the referral to the client
B. Cost of the referral service
C. Barriers to making a referral
D. Client’s eligibility for the service

24. Group with a laissez faire leader are likely to be ___.


A. More structured in format
B. Less productive
C. More productive
D. More satisfying to members

25. You have volunteered to be part of a work group whose purpose is to look at ways to prevent
medication error. You know that:
A. Teamwork enhances the probability of goal achievement
B. Group process is not important in a task
C. The leader assumes responsibility for the overall group functioning
D. The goal should be set by management

Situation - Some disasters occur relatively frequent in certain parts of the world. The Philippines is one
country that has experienced both natural disasters and human made disasters.

26. The public health nurse is aware of the importance of the need to perform roles in preparing for a
disaster. The BEST example of one of these roles would be to:
A. Assist in the prevention of injury
B. Identify disaster risks
C. Coordinate emergency care
D. Inspect cluster of houses

27. The residents and local leaders in Municipality X is mapping out potential disaster locations in the
community. This period of planning and preparation is described as:
A. Recovery stage
B. Pre - disaster stage
C. Impact stage
D. Non - disaster stage

28. The nurse provides counselling for victims of flood. This is an example of _____.
A. Primary Prevention
B. Secondary Prevention
C. Mental health care
D. Tertiary Prevention
NURSING PRACTICE I: Community Health Nursing
November 17 & 18, 2018
29. Should a terrorist attack occur, nurses need to be ready, not only in providing quality care for the victim
but ensuring their own safety as well. During a mass disaster drill stimulating a terrorist attack, the nurse
must triage numerous severely ill persons. The client who should receive PRIORITY care is:
A. Cyanotic and not breathing
B. Apneic and has an apical rate of 50
C. Having seizures and urine incontinence
D. Gasping for breathing and are conscious

30. The nurse is considering how best to educate the community about the potential threats to health from
terrorism. What are the BEST ways to accomplish this?

1. Hold public forum to educate the community about coping with psychology effects.
2. Raise community awareness about increased immigrants to the population
3. Teach the community about sealing windows and doors in the event of a chemical attack.
4. Raise community awareness about the signs and symptoms of potential biologic agents.
A. 2 and 4
B. 1 and 3
C. 2 and 3
D. 1 and 4

Situation - Public health nurses like Ms. Shalom should be knowledgeable on the importance of immunity
in the prevention of communicable diseases by building the capacity of the body to restrict harmful
microorganisms or viruses from entering it.

31. Nurse Shalom knows that the type of immunity which is longest acting is____________
A. Natural immunity
B. Passive immunity
C. Artificial immunity
D. Active immunity

32. She learned that the MOST important function of inflammation and immunity is___________
A. Preventing any entry of foreign material
B. Providing protection against invading organisms
C. Regulating the process of self- tolerance
D. Destroying bacteria before damage occurs

33. Nurse Shalom administers hepatitis B immunoglobin serum to Helen, which will provide her with
passive immunity. One advantage of passive immunity is that it______
A. Encourages the body to produce antibodies
B. Has effects that last a long time
C. Is highly effective in the treatment of disease
D. Offers immediate protection

34. Nurse Shalom explains to a mother whose ten year child just received a tetanus - toxoid injection that
the toxoid vaccine confers which of the following immunity?
A. Long lasting active immunity
NURSING PRACTICE I: Community Health Nursing
November 17 & 18, 2018
B. Lifelong natural immunity
C. Intermediate artificial immunity
D. Short - acting passive immunity

35. Nurse Shalom knows that one of the following is NOT a vaccine preventable disease, hence no
immunity can be given to a child.
A. Measles
B. Polio
C. Hepatitis B
D. Asthma

Situation - As a new public health nurses at a Rural Health Unit, Ms. Ofelia reviews basic concepts related to
her position and job.

41. Ms. Ofelia know that the PRIMARY goal of community health nursing is to ________.
A. Enhance the capacity of individuals, families and communities manage their health needs
B. Contribute to people’s well-being through sustainable development goals projects
C. Increase the productivity of the people by providing them with health services
D. Support and supplement the efforts of the medical profession in illness prevention

42. Community health nursing (CHN) is a field of nursing practice. Which best explains this statement?
A. The scope of practice depends on the health needs and problems identified.
B. The services are delivered outside of purely curative institutions.
C. The service are provided along with community health volunteers.
D. The practice is conducted in the geographical location of people.

43. Ms. Ofelia is aware that she will be dealing with population as a client. Which of the following provides
the BEST definition of a population?
A. Cluster of individuals who are at-risk of certain health conditions.
B. Group of people sharing the same geographic environment.
C. Collection of people who are beneficiaries of health services.
D. Collection of individuals who share common characteristics.

44. An example of an aggregate – a group of people with common characteristics or population that Ms.
Ofelia should consider in program planning would consist of ______.
A. Patients confined in a district hospital
B. Christians in the community
C. Young children and adolescents attending school
D. People playing games three times a week in a senior citizen club

45. Which Nursing activity is unique to community/public health nurses like Ms. Ofelia?
A. Care for sick and dying
B. Promote livelihood projects assist poor families
C. Home visits to family clients referred by the health center physician
D. Focus on vulnerable groups in the community
NURSING PRACTICE I: Community Health Nursing
November 17 & 18, 2018
Situation – In public health, quality improvement (QI) techniques are used to improve performance and
service delivery.

46. PDCA is the basic structure for most quality improvement (QI) process. In public health, PDCA is focused
on activities that are responsive to community needs and improving population health. PDCA stands for:
A. Perform-Document-Correct-Act
B. Perform-Do-Check-Analyze
C. Plan-Document-Check-Analyze
D. Plan-Do-Check-Act

47. There are several approaches and tools in (QI). One of these is the Fishbone diagram, developed by K.
Ishikawa. Which BEST describes the Fishbone diagram? It is ____.
A. A cause-and-effect diagram
B. A wholistic approach in problem identification
C. Used in interview sessions
D. Useful in focusing on problem symptoms

48. As a QI tool, the Gantt chart is useful for planning and scheduling projects. Which of the following
statements is INCORRECT about the GANTT chart? It ______.
A. Provides a graphical illustration of resources
B. Plans the order in which you’ll complete tasks
C. Tracks specific tasks in a project.
D. Helps assess how long a project should take

49. The steps in conducting quality improvement consist of the following: Which is the correct sequence in
the QI process?

1. Identify and select strategies


2. Evaluate QI outcomes
3. Implement solution in small scale
4. State the problem
5. Expand scope of QI throughout the organization
A. 1, 2, 3, 5, 2
B. 2, 1, 5, 3, 4
C. 4, 2, 1, 3, 5
D. 1, 4, 5, 3, 2

50. Client satisfaction survey is an important and commonly used indicator for measuring the quality in
health in health care. In the context of public health, which is NOT a purpose of satisfaction surveys? To
______.
A. Serve as essential sources of information for identifying gaps
B. Directly address the health problems presented
C. Present the likelihood of continuing client-provider-agency relationships
D. Provide data about health-seeking behaviors
NURSING PRACTICE I: Community Health Nursing
November 17 & 18, 2018
Situation – Promoting mental health is equally important in promoting physical health. Unfortunately,
metal health has been largely neglected.

51. Public Health Nurse Victoria is concerned about the impact of metal health on the community. She is
preparing for a presentation about mental health in the Philippines. Which of the following information
should be included?
A. Mental illness is an increasing problem among the upper class.
B. The incidence of depression is expected to decrease within the next 10 months. increase
C. Almost one per 100 household has a member with mental disability.
D. Post traumatic stress disorder is the most common of anxiety disorders.

52. The PRIMARY reason that mental illness often results in worsening of physical health problems is
because of the _______.
A. Loss of cognitive function
B. Feelings of inadequacy
C. Inability or lack of motivation for effective self-care
D. Side effects of mental health modification

53. Nurse Victoria shared the statistics that intentional self-harm was found to be the 9th leading cause of
death among 20-24 years old Filipino adults. Increasing number of mental health problems may be related
to _______. (Select all that apply)

1. Poverty
2. Age of the parents
3. Family unemployment
4. Changes in family structure
5. Lost of community social support
A. 1, 2, and 5
B. 1, 3, 4, and 5
C. 1, 2, 3 and 4
D. 1, 2 and 3

54. Nurse Victoria noted that there were number of poor older women, living alone, who are suffering
from depression. Her FIRST goal is to ______.
A. Improved quality of life
B. Provide recreation and entertainment
C. Reduce suicide risk
D. Improve level of function

55. In promoting mental health, Nurse Victoria is cognizant of her nursing responsibilities. These include
the following, EXCEPT:
A. Teach parents the importance of community of providing emotional support to their children.
B. Increase general knowledge of the community on knowledge on mental hygiene.
C. Help the community address factors that promote mental well-being.
D. Refer at once cases of bizarre behaviors to a psychiatrist.
NURSING PRACTICE I: Community Health Nursing
November 17 & 18, 2018
Situation – Public Health Nurse Naomi applies model and concepts in epidemiology relevant to her nursing
practice.

56. Nurse Naomi encourages that parents to have their kinds receive measles vaccination stating that those
who are susceptible may have serious consequences. She also tells them of the benefits of the
immunization. Most parents decide to have the immunization. Which of the following models is used to
guide parent’s decision?
A. Pender’s Health Promotion Model
B. Precede-Proceed Model
C. Reasoned Action Model
D. Health Belief Model

57. In this situation, Nurse Naomi notes that the agent in the epidemiolocal triad that brings about measles
is the ______.
A. Genetic susceptibility
B. School
C. climate
D. virus

58. Nurse Naomi has to be alert about the time interval between the invasion by an infection agent and the
appearance of the first sign or symptom of the disease. This is referred to as ______.
A. Generation time
B. Incubation period
C. Communicability period
D. Virulence

59. Should an outbreak of a gastrointestinal illness resulting from a food-borne pathogen happens, Nurse
Naomi would likely to think that this is a/an:
A. attack rate
B. secular trend
C. point epidemic
D. event-related cluster

60. Which of the following denotes that resistance of an entire community to an infectious agent as a result
of the immunity of a large proportion of individuals to the agent?
A. Active immunity
B. Passive immunity
C. Natural immunity
D. Herd immunity

Situation – A public health nurse is giving orientation to three beginning nurses who are hired by the local
government unit under the Nurse Deployment Programs.

61. In the first session, the nurse presented the organization chart of the Rural Health Unit. Which of the
following aspects of organizational structure is illustrated in the chart?
NURSING PRACTICE I: Community Health Nursing
November 17 & 18, 2018
1. Type of work being done
2. Extent of coordination to be done
3. Levels of management
4. Line and staff relationship
A. 2, 3, 4
B. 3 and 4
C. 1, 3, 4
D. 1 and 3

62. The PHN wants to emphasize to the new nurses it is important to observe professional accountability
which is taking responsibility for one’s actions. She explains that accountability can be done by:

1. Asking assistance from the doctor


2. Performing nursing tasks in a safe manner
3. Reporting and documenting assessment and interventions
4. Evaluating client’s response to nursing care given
5. Evaluating the care when there is complaint by the client
A. 1, 2, 3
B. 1, 3, 5
C. 2, 4, 5
D. 2, 3, 4

63. Accountability also includes a commitment to continuing education to stay current knowledgeable.
According to the Guidelines Implementing Continuing Professional Education for Nurses (CPE), from which
of the following can a nurse earn credit units?

1. Participation in seminars
2. Engagement in a research project
3. As peer reviewer
4. Innovations
A. 2 and 3 only
B. 1, 2, 3, 4
C. 1 and 2 only
D. 1, 3, 4

64. The PRIMARY purpose of requiring continuing education is to?


A. Qualify for work abroad
B. Improve nursing competency after registration
C. Comply with the Board of Nursing requirements
D. Ensure legal practice

65. To better address emerging public health issues, a public health nurse enrolls in a course in:
A. Ethics
B. Research
C. communication
D. Leadership
NURSING PRACTICE I: Community Health Nursing
November 17 & 18, 2018

Situation – As a public Health nurse, you are aware that many members of the community are poor and
underserved, affected their health status.

66. In the Philippines, poverty remain a challenge the following are some facts on poverty based on the
latest (2015) statistical report. Which statement is NOT included?
A. 28 percent of the country’s 97 million people live below the poverty line
B. Fifteen million Filipinos will rise above poverty in 2020.
C. Farmers, fishermen and children consistently posted the highest poverty incidence.
D. More than 12 million Filipinos are living in extreme poverty.

67. In identifying factors that contribution to poverty, homelessness, and poor health of a deprived
segment of a population, which of the following should you assess?
A. Mental illness and community support
B. Provision of social support and basic services
C. Mobility and age of family members
D. Neighborhood environment and sanitation

68. You have noticed that the population in a poor community has increased including the rise of pregnant
women who are undernourished. You coordinate with the local social welfare department to provide food
that are nutritionally adequate. This is considered:
A. Health promotion
B. Tertiary prevention
C. Primary prevention
D. Secondary prevention

69. In another segment of the community, you realize that there are young adult members who have poor
nutritional habits. You know that these are risk factors for diabetes and cardiovascular disease. These risk
factors can affect their ______.
A. Future potential to change their dietary habits
B. Ability to obtain health services.
C. Ability to network social support systems
D. Health status and employment potential

70. You have learned that people with mental and psychosocial disabilities are a vulnerable group. Who are
restricted in their ability to access essential health and social care. In your advocacy, you will include the
following messages, EXCEPT:
A. Persons with mental and psychosocial disabilities should be confined in an institution
B. Employment and job opportunities must be created for people with mental and psychosocial disabilities.
C. Mental health should be included in services during and after emergencies and disasters.
D. Mental health services should be integrated systematically into all health services including primary level
care.

Situation – The Department of Health reported that Filariasis is endemic in 45 out of


78 provinces in the Philippines. You are assigned in the province of Saranggani, one of the endemic areas.
You know that Filariasis carries a social stigma owing to the disfiguring appearance brought by the disease.
NURSING PRACTICE I: Community Health Nursing
November 17 & 18, 2018

71. You call that Filariasis is caused by parasitic nematode known as ________.
A. Wucheria bancrofti
B. Necator americanus
C. S. haematobium
D. Plasmodium falciparum

72. The disease is transmitted to a person through bites of an infected female mosquito, known as _____.
A. Aedes aegypti
B. Culex
C. Anopheles
D. Aedes poecillus

73. You receive a 36-year-old man who was brought by his wife to be tested for Filariasis. The most likely
diagnostic test that he will undergo is ______.
A. Immunochromatographic test (ICT)
B. Rumpel Leads Test
C. Urinalysis
D. Stool examination

74. A client in the ACUTE stage of Filariasis will include which of the following clinical findings?
A. Hydrocele, lymphedema, elephantiasis
B. Lymphadenitis, lymphedema and orchitis
C. Orchitis, hydrocele, elephantiasis
D. Lymphangitis, lymphadenitis, epididymitis

75. Effective methods that the government would likely to pursue to eliminate Filariasis in the country are
all of the following, EXCEPT:
A. Vaccination off all susceptible in risk areas and high-risk populations.
B. Pursue annual mass drug administration using two drugs in all endemic areas for five consecutive years.
C. Intensify health information and advocacy campaigns in its prevention, control and elimination.
D. Intensify environmental sanitation such as proper drainage and cleanliness of surroundings.

Situation – In any setting of nursing practice including community health, records management such as
documentation and record keeping, is important.

76. Documentation is a critical component to the delivery of healthcare. It is a tool which serves many
purposes. Which of the following is NOT included?
A. Create a permanent record for the patient’s future care
B. Provide material for discussion
C. Plan and evaluate a patient’s treatment
D. Ensures continuity of care

77. When recording the home visit, it is important for the public health nurse to _____.
A. Document the visit only when there are significant changes
B. Follow the agency format for recording and documentation
NURSING PRACTICE I: Community Health Nursing
November 17 & 18, 2018
C. Complete the charting every Friday of the week
D. Use phrases in outline form

78. The nurse should document intelligently and clearly. There are ways to help protect against an
allegation of falsifying a medical record. These are the following:

1. Date, time, and sign every entry.


2. Make entries soon after care is given.
3. Written legibly.
4. Be thorough, accurate, and objective.
5. Use only approved abbreviations.
A. 1, 2, 3, 4, and 5
B. 1, 3, and 5
C. 2, 3 and 4
D. 2, 4, and 5

79. Public health nurses must know that all records have a life cycle and retention scheduling. Clinical
records must be retained in the health care facility for:
A. Anytime with client’s permission
B. Ten years from last date of service
C. Two years from date of client’s last visit
D. Five years from date of client admission

80. Controlled Substances such as drugs dispensed and administered, order and inventory records must be
kept in the health care facility for how many years?
A. Six months
B. One year
C. Five years
D. Three years

Situation – Avelina has recently passed the National Licensure Examination for Nurses. She intends to apply
in the Nurse Deployment Project (NDP) of the Department of Health.

81. The NDP is open to all nurses who fulfill the following qualifications, EXCEPT:
A. Willing to undergo recruitment and selection process.
B. Possesses an official and validation PRC – Professional Identification Card
C. Must be at least 21 years old at the time of application.
D. Physically and mentally fit as shown in the medical certificate.

82. The NDP has been designed by the Department of Health PRIMARY to ______.
A. Improve local health systems in support of Universal Health Care
B. Augment staff of rural Health units
C. Provide experience to new nurses in terms of work realities
D. Prevent nurses from seeking jobs abroad
NURSING PRACTICE I: Community Health Nursing
November 17 & 18, 2018
83. Under this program, Avelina will be hired on a Contract of Services status. She will receive a monthly
Salary (PhP) of:
A. 13,000
B. 18,000
C. 15,000
D. 22,000

84. Nurse Avelina knows that as NDP nurse, she may be assigned in any of the following areas:

1. Rural Health Units


2. Barangay Health Stations
3. Disaster-prone areas
4. Level 1 LGU Hospitals
5. DOH Hospitals
6. Birthing homes
A. 2, 4, and 6
B. 1, 3, and 4
C. 2, 3, 5 and 6
D. 1, 2, and 5

85. The NDP is a partnership between the DOH and other government agencies and organizations. Which
of the following institutions is NOT included?
A. Department of Social Welfare and Development
B. Professional Regulatory Board of Nursing
C. Local Government Units
D. Philippine Nurses Association

Situation – Risk factors include genetic and physiological factors, age, physical environment and lifestyle.

86. The presence of any of these risk factors means that ______.
A. Risk modification will have no effect on disease prevention
B. The chances of getting the disease are increased
C. A person with risk factors can get the disease
D. The disease is guaranteed not to get disease

87. A nurse includes well-being in her definition of health. What will the focus be when the nurse provides
care to a family confronting the imminent death of a family member?
A. Reinforcing the need for the whole family to adapt to the impending death
B. Sharing the nurse’s knowledge on the client’s symptoms of grief
C. Discussing the perceived meaning of life with the family
D. Administering pain medication so that the client can be kept comfortable

88. In the care of families, crisis intervention is an important part of _____.


A. Secondary prevention
B. Tertiary prevention
C. Primary prevention
NURSING PRACTICE I: Community Health Nursing
November 17 & 18, 2018
D. Health promotion

89. The Nurse asks Dante, who is being admitted in a district hospital, with uncontrolled diabetes mellitus,
about his employment status. She knows that _____.
A. A person’s compliance is affected by one’s affected economic status
B. Dante’s employment will cause complication to his diabetes
C. External variables have little effected on compliance
D. Diabetes mellitus will affect his client’s work condition

90. During a home visit, a member of the family who has been using marijuana for three years tells the
nurse that he wants to be “detoxified.” It is important for the nurse to ____.
A. Instruct the patient to change his lifestyle
B. Tell the patient that relapses are not tolerated
C. Identify the patient’s stage of change
D. Realize that the patient is ready for change

Situation – As a public health nurse, you may encounter barriers or challenges in communication. Thus, it is
important that you maintain your professional demeanor.

91. During a support group meeting, Nurse Donato, in a teasing manner, has made several provocative
about your appearance and behavior as a group leader. Select your MOST appropriate response.
A. “Donato, see after this meeting.”
B. “Donato, what you are saying is inappropriate.”
C. “Donato, you are excused from the group.”
D. “What do you think Donato is trying to tell us?”

92. “The dress code in this facility does not include wearing blue jeans while on duty.” This statement is an
example of using which step in giving constructive criticism?
A. Listing consequences
B. Expressing empathy
C. Describing the behavior
D. Stating expectations

93. Which of the following is an example of a barrier to successful negotiation?


A. Understanding other’s perspective
B. Becoming emotional
C. Viewing negotiation as collaborative
D. Avoiding phrases that are coercive

94. In one of your meeting you reminded the health center staff that there are quite a number of members
in the community who have not reached primary school. Which of the following is the BEST communication
intervention for a client who is illiterate?
A. Use symbols and images
B. Use touch with speech
C. Personalize speech by using first name
D. Speak loudly and clearly
NURSING PRACTICE I: Community Health Nursing
November 17 & 18, 2018

95. You also emphasized that, “communication must be culturally competent to be effective”. Which of the
following BEST reflects this statement?
A. Listen actively to what is said.
B. Reflect on the meaning of the message.
C. Use simple, direct words.
D. Provide an appropriate environment.

Situation – Ms. Nenita will be involved in an interdisciplinary research to be conducted by the city Health
Office to address health problems of the population covered. To prepare for this tasks, she reviews her
knowledge on the research process and research design.

96. Which of the following study designs that uses information on current health status, personal
characteristics, and potential risk factors will be appropriate?
A. Case-control
B. Ecological
C. Cohort
D. Cross-sectional

97. The type of epidemiologic study that is used to describe a group of persons enrolled in a study who
share some characteristic of interest and who are followed over a period of time to observe some health
outcome is a(n):
A. Cross-sectional study
B. Experimental study
C. Cohort study
D. case control study

98. The research team will use summary indicators of health as method to depict health status. What is this
approach called?
A. Analytic
B. Holistic
C. Descriptive
D. Evaluative

99. Ms. Nenita is specifically assigned to talk individually with community leaders like the primary grade
teacher and the barangay official in charge of the health committee. Which data collection method will the
nurse use?
A. Key informant interview
B. Community mapping
C. Participant observer
D. Social survey

100. Ms. Nenita suggested to use an analytic approach in their research. Which of the following statements
BEST describes the analytic approach?
A. Focuses on the community’s health problems and issues
B. Utilizes health indicators to define population health
NURSING PRACTICE I: Community Health Nursing
November 17 & 18, 2018
C. Views factors that influence health and by which interventions to improve health are directed
D. Addresses the social determinants of health to solve community’s problem

ANSWER KEY FOR NURSING PRACTICE I

1.B 11.C 21.A 31.D 41.A 51.A 61.A 71.A 81.C 91.B
2.C 12.D 22.D 32.B 42.C 52.C 62.D 72.B 82.A 92.D
3.D 13.B 23.D 33.D 43.B 53.B 63.B 73.A 83.B 93.B
4.B 14.D 24.B 34.A 44.C 54.C 64.B 74.D 84.D 94.A
5.B 15.A 25.A 35.D 45.D 55.D 65.B 75.A 85.A 95.D
6.B 16.C 26.B 36.- 46.D 56.D 66.A 76.B 86.B 96.D
7.D 17.B 27.B 37.- 47.A 57.D 67.B 77.B 87.A 97.C
8.C 18.B 28.A 38.- 48.A 58.B 68.C 78.A 88.A 98.A
9.C 19.C 29.D 39.- 49.C 59.C 69.D 79.B 89.D 99.A
10.D 20.C 30.D 40.- 50.B 60.D 70.A 80.C 90.D 100.B
NURSING PRACTICE I: Community Health Nursing
November 17 & 18, 2018
NURSING PRACTICE II: Care of Healthy/At Risk Mother and Child
November 17 & 18, 2018
Situation – Headnurse Mila wishes to successfully change the way her registered nurses, nursing assistants,
and other nursing staff in providing emotional support to the ward’s patients.

1. In conducting quality improvement, Nurse Mila should look into which of the following as her basis,
EXCEPT:
A. Patient’s complaint data
B. Patient’s satisfaction survey
C. Incident reports
D. Patient’s chart

2. The FIRST step in the Quality Improvement process is which of the following?
A. Assess progress.
B. Examine the data.
C. Confirm the existence of the problem by gathering data.
D. Set goals.

3. The BEST quality improvement intervention that Nurse Mila can do is which of the following?
A. Maintain ongoing informal communication with staff.
B. Analyze patients complaint record
C. Interview staff individually and in group
D. Recommend increase in salary of nursing staff.

4. Some of the nursing staff seemed to have a sense of entitlement according to nursing department
leadership. They behaved as though they had unlimited job security (because of the strength of the nursing
unions). Which of the following action is the BEST?
A. Develop a documentation-and-tracking process for the observations
B. Revise job descriptions and performance for all nursing staff to include the new general care guideline
and highlight emotional support.
C. Continue to work on increasing staff accountability for service behaviors.
D. Develop new evidence-based general care guidelines for nurse-patient interactions.

5. Which of the following requires quality improvement? Select all that apply.

1. Patient’s fall/injury
2. Infection
3. Safety
4. Pressure ulcers
A. 1, 2, & 4
B. 3 only
C. C. 1 & 4
D. D. 1, 2, 3, & 4

Situation - Nurse Crissel was talking about HIV – AIDS to a group of teenagers. Here are some questions
which were asked by them during the open forum.
NURSING PRACTICE II: Care of Healthy/At Risk Mother and Child
November 17 & 18, 2018
6. What are some of the general symptoms the PRIMARY HIV infection? The nurse enumerated the
following. Which among these are CORRECT?

1. Fatigue
2. Headache
3. Fever
4. Sore throat
5. Cough
6. Dyspnea
A. 1, 2, 3, 5, 6
B. 1, 2, 4, 5,
C. 1, 2, 3, 4
D. 2, 3, 4, 6

7. When can AIDS be manifested? The nurse answer was. “It can be as early as _______.”
A. 1 year or as late as 2 years
B. 2 years or as late as 10 years
C. 1 year
D. 6 months

8. Nurse Crissel also asked the participants if they got to know the transmission of HIV based from her
lecture? Which is Not correct?
A. Accidental blood exposure
B. Kissing
C. Unprotected sex
D. Mother to child transmission

9. On the question as to which of the following are the effects of AIDS on pregnancy, one teenager cited a
wrong answer which was ____.
A. Mild weight loss
B. Prematurity
C. Repeated abortion
D. Infertility

10. HIV transmission from mother to infant occur at post-natal period during _____.
A. Bathing
B. Washing of vagina
C. Bottle feeding
D. Breastfeeding

Situation – Ethics is a field of moral science which deals with the morality of human acts. Registered nurses
must be aware that their actions have professional and ethical dimensions. They should strive to perform
their work to the best interest of all concerned.

11. Which of the following is NOT aligned to ethics in nursing? A Nurse ______.
A. Has the freedom to do what he likes sans responsibility
NURSING PRACTICE II: Care of Healthy/At Risk Mother and Child
November 17 & 18, 2018
B. Is obliged to avoid what is wrong and do what is good
C. Is a person capable of knowing what is right or wrong
D. Should have a sense of accountability for his actions

12. A professional nurse has a duty to know and respect the Patient’s Bill of Rights. Which of the following
is NOT included in the Patient’s Bill of Rights. The right to/for ______.
A. A considered and respectful care.
B. Privacy and confidentiality
C. A complete and current information about his illness
D. Expect continuity of care from discharge to full recovery

13. What bio-ethical principle is violated by a nurse if he provides his patient fraudulent information about
his diagnosis and prognosis?
A. Justice
B. Autonomy
C. Beneficence
D. Veracity

14. Which of the following directly VIOLATES the Patient’s Bill of Rights?
A. Informing patients about the billing policies of the hospital
B. Disclosing the HIV result to members of the patient’s family.
C. Honestly telling the patient about his current condition
D. Immediately referring results of laboratory to the physician.

15. Autonomy is the prerogative of the patient to give consent or refusal of treatment with the EXCEPTION
of which of the following situations?
A. Erroneous belief of a head of a church
B. Negative effect of superstition
C. Near death
D. Peer pressure

Situation – Thelma, a mother of a 6-year-old boy Marco has arrived at school to take her child home
because the school Nurse James has verified that he has an inflamed throat. Nurse James urges his mother
to seek treatment because if the causative agent is beta-hemolytic streptococcus, he may develop a
disorder characterized by inflamed joints, fever, and the possibility of endocarditis.

16. Which of the following would be the possible disorder?


A. Tetanus
B. Rheumatic fever
C. Scarlet fever
D. Influenza

17. Alice’s mother question whether her other children can catch the same disease. Which of the following
should be the nurse’s response? *
A. Your other children should be taking antibiotics to prevent them from catching the same disease.
NURSING PRACTICE II: Care of Healthy/At Risk Mother and Child
November 17 & 18, 2018
B. It is caused by an autoimmune reaction and is not contagious.
C. You appear concerned that your daughter’s disease is contagious.
D. The fact that you brought Alice to the hospital early enough will decrease the chance for her siblings
getting it.

18. In addition to carditis, which of the following should the nurse assess the child?
A. Oliguria and edema
B. Malabsorption and diarrhea
C. Arthralgia and low grade fever
D. Bronchitis and pneumonia

19. After throat swab for culture and sensitivity, the nurse would expect the physician to prescribe an
appropriate antibiotic. Which of the following is the purpose of this medication? It is to prevent _______.
A. Recurrence
B. Transmission
C. Inhalation
D. Inflammation

20. If left untreated, such condition can progress to which of the following complication?
A. Kidney failure
B. Left-sided heart failure
C. Right-sided heart failure
D. Angina pectoris

21. When planning a teaching program for a child who has recently been diagnosed with type 1 diabetes,
what will be the nurse’s FIRST concern for the Adel and her parents? To let them ______.
A. Assess their own feelings about diabetes
B. Learn how to monitor blood glucose level
C. Understand why activities must be limited
D. Learn how to administer insulin injections

22. An evening snack is planned for Adel receiving NPH (Novolin N) insulin INTERMMEDIATE. This will
provide _____.
A. Encouragement for the child to stay on a diet
B. Calories to help the child gain weight
C. Nourishment to counteract late insulin activity
D. Energy for immediate utilization

23. When teaching about insulin and its potential for hypoglycemia, the nurse should include that its PEAK
EFFECT occurs in which number of HOURS?
A. 1 to 2
B. 5 to 10
C. 4 to 12
D. 2 to 4

24. When teaching Adel on dietary management, what should the nurse emphasis MOST?
NURSING PRACTICE II: Care of Healthy/At Risk Mother and Child
November 17 & 18, 2018
A. Food in the form of concentrated glucose should be available all the time.
B. Meals should be preferably prepared and eaten at home
C. Food should be weighed on a gram scale all the time.
D. Meals should be prepared separately from the rest of the family.

25. At 7 AM, the nurse receives the information that Adel has a 6 AM fasting blood glucose level of 180
mg/dL. What should be her PIORITY nursing action?
A. Inform Adel that a complex carbohydrate such as cheese should be eaten.
B. Encourage Adel to start exercising and to continue for 5 minutes.
C. Tell Adel that the prescribed dose of regular insulin should be administered.
D. Ask Adel to obtain again an immediate glucometer reading.

Situation – To ensure patient safety at all times, personal and professional growth of nursing is a necessity
than a choice. Nurse Florence is a beginning nurse in one of the hospitals in the suburb. During the
Orientation Program, the need to enhance her skills, knowledge and attitude was emphasized.

26. “Commitment to continual learning and active participant in the development and growth of the
profession are commendable obligation” is contained in which of the following?
A. Code of Ethics
B. RA 10912
C. RA 7164
D. RA 9173

27. Which law declares that the policy of the State is to promote and upgrade the practice profession in the
country?
A. RA 7164
B. Ra 0173
C. code of Ethics
D. RA 10912

28. Nursing programs that are based on needs assessment and needs analysis and should be offered free is
which of the following?
A. Continuing Development Program
B. In-service Training Program
C. Nursing Education Program
D. Post-Graduate Program

29. In three year’s time, Nurse Florence needs to attain certain number of Continuing Professional
Development units to be able to renew her ______.
A. Philippine Nurses Association membership
B. Professional License
C. Professional Identification Card
D. Professional Registration

30. Nurse Florence was asked by her Head nurse why she requested permission to enroll in the Graduate
Program for the second semester. Her reply should be, EXCEPT:
NURSING PRACTICE II: Care of Healthy/At Risk Mother and Child
November 17 & 18, 2018
A. To have an impressive resume to enable her to be competitive
B. Connect with people professionally
C. Invest for the future
D. Pursue her interest in Pediatric Nursing in more depth.

Situation – Nurse Carmi made a study on domestic violence against women in a certain province in Luzon.

31. Since she wanted to capture the essence and emotion of the victims she choose to use the qualitative
design. Which among the statements below is CORRECT about qualitative design?
A. It commences study at present but consummates at any future time.
B. It allows estimation of relationship between studied variables.
C. It provides insights into attitudes, beliefs, motives and behaviors of target population.
D. It tests result through numerical data and subjects them to statistical analysis.

32. Nurse Carmi plans to gather 6 participants who are victims of domestic violence where she will base
her questionnaires. What technique in qualitative research is applicable to this plan?
A. Data triangulation
B. Focus group discussion
C. Interview
D. Case study

33. To have a better analysis and interpretation of finding, Nurse Carmi reviewed and compared them with
other findings of previous researches on the same topic. This is done MAINLY by doing back to which part
of the study? The __________.
A. Statement of the problem
B. Related literature and studies
C. Conceptual framework of the study
D. Theoretical framework of the study

34. Nurse Carmi finally decided to make an in depth study of ONLY ONE SUBJECT of domestic violence.
What design will she use?
A. Causality Design
B. Predictive Correlational Design
C. Descriptive Correlational Design
D. Descriptive Case Study Design

35. In as much as the research may have some risks on the part of the subject who is 21 years old, Nurse
Carmi must assure the subject of her anonymity, confidentiality and respect for her human rights. ONE
ABSOLUTE WAY is to get an informed and written consent from whom?
A. Parents
B. Guardians
C. Husband
D. Subject herself
NURSING PRACTICE II: Care of Healthy/At Risk Mother and Child
November 17 & 18, 2018
Situation – Hemophilia is a rare blood disorder that affects the body’s clothing factors. This may result to
incapacitation complications to children affected with this disorder. A 3-year-old child Billy was diagnosed
with hemophilia.

36. When counselling parents of a child who has recently been diagnosed with hemophilia, what must
Nurse Thelma KNOW about Billy’s condition whose father is normal and the mother is the carrier?
A. It is likely that all sons are affected.
B. There is a 50% probability that sons will have the disease.
C. Every daughter is likely to be a carrier.
D. There is a 25% chance a daughter will be a carrier.

37. Billy has slipped on the ice and bumped his knee. Which among the following should Nurse Thelma
prepare to administer, as per doctor’s order? Intravenous infusion of ______?
A. Cryoprecipitate
B. Factor VIII
C. Factor X
D. Desmopressin (DDAVP)

38. Nurse Thelma is providing home care instructions to the mother of Billy. Which of the following
complications should Nurse Thelma tell the mother, should repeated bleeding continues?
A. Leukemia
B. Hemarthrosis
C. Ecchymosis
D. Hematoma

39. A nurse analyzes the laboratory results of Billy. The nurse understands that the MOST likely ABNORMAL
finding in Billy is which of the following?
A. Partial thromboplastin time
B. Hemoglobin level
C. Hematocrit level
D. Platelet count

40. The nurse is planning a meal that would provide IRON for a child with bleeding disorders. Which dinner
menu would be the BEST?
A. Chicken nuggets, macaroni, peas, cantaloupe, milk
B. Fish sticks, French fries, banana, cookies, milk
C. Ground beef patty, lima beans, wheat roll, raisins, milk
D. Peanut butter and jelly sandwich, apple slices, milk

Situation – Conchita was diagnosed with gestational diabetes. She is 34 years old and is on her 26 weeks
Age of Gestation.

41. Gestational diabetes is said to exist in pregnancy because of the diabetogenic effect of what hormone
secreted by the placenta?
A. Human placental lactogen
B. Human chorionic gonadotropin
NURSING PRACTICE II: Care of Healthy/At Risk Mother and Child
November 17 & 18, 2018
C. Estrogen and progesterone
D. Relaxin

42. Which of the following infection will mothers with diabetes have FREQUENTLY?
A. Moniliasis
B. Herpes zoster
C. psoriasis
D. herpes simplex

43. In 36 weeks, amniocentesis was orders by the doctor for this patient. What is the purpose of this
procedure? To detect/determined ______.
A. An increase level of bilirubin
B. Genetic abnormalities
C. Down syndrome
D. Fetal lung maturity

44. Aside from an endocrinologist, to whom should the nurse also plan to refer the patient to?
A. General practitioner
B. Neurologist
C. Surgeon
D. Hyperglycemia

45. What is the PRIMARY effect to the newborn of a diabetic mother?


A. Hypoglycemia
B. Anemia
C. hyperinsulinism
D. hyperglycemia

Situation – Patient Clara visits her obstetrician for a positive pregnancy test. The nurse in the clinic took her
history. Clara reported that she has been pregnant four times before. She has two children at home, one of
whom was born at 32 weeks gestation. She lost a set of twins at 14 weeks and another baby at 12 weeks.

46. In taking the Obstetrical score, how will Nurse Merlita record Gravida, Para and (TPAL)?
A. G5, P4, T1, P1, A2, L2
B. G5, P2, T1, P1, A2, L2
C. G3, P2, T1, P2, A2, L1
D. G4, P4, T2, P3, A3, L2

47. When questioned, Clara admitted that she sometimes has several glasses of wine with dinner. Her
alcohol consumption puts her fetus at risk for which condition?
A. Alcohol addiction
B. Anencephaly
C. Learning disability
D. Down syndrome
NURSING PRACTICE II: Care of Healthy/At Risk Mother and Child
November 17 & 18, 2018
48. To help determine whether Clara is at risk for a toxoplasmosis infection, what will the nurse ask the
client?
A. “Have you ever had osteomyelitis?”
B. “Have you recently has a rubeola vaccination?”
C. “Do you have any cats at home?”
D. “Do you have any birds at home?”

49. Clara asks Nurse Merlita whether she can take castor oil for her constipation. How should the nurse
respond? “No”. It can ______.”
A. Lead to increased absorption of fat-soluble vitamins
B. Promote sodium retention
C. Produce an adverse effects
D. Initiate premature uterine contractions

50. Clara also admitted to Nurse Merlita that she also use cocaine at least once per day, skipping meals at
times. Which nursing diagnosis is MOST appropriate for her?
A. Activity intolerance related to decreased tissue oxygenation.
B. Risk for infection related to metabolic and vascular abnormalities.
C. Impaired gas exchange related to respiratory effects of substance abuse.
D. Impaired nutrition: less than body requirements related to limited food intake.

Situation – Nursing assessment of individuals across the life span requires a nurse to be familiar with the
physiological, cognitive, and psychosocial changes that occur during each stage of development.

51. The factors that affect the ability of individuals to cope are______. Select all that apply.

1. Level of development
2. Coping skills
3. Previous experiences with illness and hospitalization
4. Seriousness of diagnosis
A. 2, 3, and 4
B. 1, 2 and 4
C. 1, 2, 3, and 4
D. 1, 2, and 3

52. Appropriate nursing diagnoses for patients with developmental problems are, EXCEPT:
A. Delayed growth and development
B. Compromised family Coping
C. Impaired social interaction
D. Altered Sleeping Pattern

53. Patient’s participation in the planning phase of the nursing diagnosis depends on ______. Select all that
apply.

1. Development Status
2. Psychological condition
NURSING PRACTICE II: Care of Healthy/At Risk Mother and Child
November 17 & 18, 2018
3. Level of intelligence
4. Comprehension of his disease
A. 2 & 3
B. 1 & 2
C. 1, 2, 3, & 4
D. 3 & 4

54. When a 15-year-old female patient is so engrossed and worries as to what her friends think about her,
she is categorized to be in the psychosocial development stage. Nurse Nene needs to inform the mother
that this is a normal behavior. In what stage of the psychosocial development is the patient in?
A. Identity versus Role Confusion
B. Autonomy versus Doubt and Shame
C. Industry versus Inferiority
D. Initiative versus Guilt

55. The Headnurse of Pediatric Ward asked a staff nurse who is assigned to a 3 month old infant. At what
age will her patient start to drink from a cup? The staff nurse’s answer should be _____.
A. 12
B. 24
C. 9
D. 5

Situation – Nurse Katrina admitted a laboring primigravida, Yolanda, brought by an anxious taxi driver.

56. Nurse Katrina calms down the similarly anxious patient by telling her that the DEFINITIVE sign that labor
about BEGIN is _____.
A. Rupture of bag of water
B. Lengthening of cord
C. Initiation of contraction
D. Bulging of perineum

57. The discomfort expected with TRUE LABOR contractions is MOST accurately described by which of the
following? Discomfort ________.
A. Begins in the lower back, abdomen and then radiates over the entire abdomen
B. Begins in the fundus and then radiated downward to the cervix
C. Radiates from the umbilicus to the iliac region then to the lower extremities
D. Centers in the fundus of the uterus during the entire contraction.

58. Nurse Katrina completed the vaginal examination on the patient. She recorded: 75%, 7cm, 0. Which of
the following is a CORRECT interpretation of the date?
A. Effacement is 5 cm complete.
B. Fetal presenting part is engaged.
C. Dilatation 75% is completed.
D. Acceleration phase of the first stage of labor is about to begin.

59. Nurse Katrina would further estimate that the above data means the cervix is _______ of its length.
NURSING PRACTICE II: Care of Healthy/At Risk Mother and Child
November 17 & 18, 2018
A. ¾
B. 1/3
C. ¼
D. ½

60. When does the membranes of a patient in labor MOST likely to rupture?
A. During the perineal phase of the second stage of labor as the head of the body crowns.
B. During the active phase of the first stage of labor, usually at the peak of a contraction.
C. During the latent phase of the 1st stage of labor between contractions.
D. Just immediately before the labor begins.

Situation – Nurses Jessy and Karyl are assigned in high risk maternity ward. Here are some cases which they
handle for the morning shift.

61. Mrs. Purisima, 38 weeks generation is admitted at the High Risk Ward. She was admitted due to labor
pains with cervix of 6 cm. BP was 160/110mmHg. She is given MgSo4 per IV. How will Nurse Jessy
determine that the MgSo4 therapy has been effective?
A. Labor pains decreased.
B. There is clonus of the ankle.
C. BP is maintained.
D. Seizures are prevented.

62. What would Nurse Jessy monitor to suspect MgSo4 toxicity on Mrs. Purisima?
A. Tingling in the toes.
B. Rapid pulse rate and hyperventilation.
C. Decreased deep tendon reflexes.
D. Hypertension

63. If Mrs. Purisima begins to exhibit signs of labor after eclamptic seizure, what will Nurse Jessy anticipate
to occur?
A. Severe infection.
B. Abruptio placenta
C. Uterine atony.
D. Placenta accrete.

64. Nurse Karyl is assisting an obstetrician in the care of a pregnant woman in labor with herpes simplex
viral infection. The nurse would anticipate which management of the condition?
A. Administration of anti-viral drug.
B. Vaginal douche.
C. Hot sitz bath TID.
D. Preparation for Ceasarian section.

65. One of the patients of Karyl has been exhibiting signs and symptoms of Rheumatic Heart Disease (RHD).
These are the following, EXCEPT:
A. Fever
B. Abnormal pulse rhythm
NURSING PRACTICE II: Care of Healthy/At Risk Mother and Child
November 17 & 18, 2018
C. Petechia
D. Heart murmurs

Situation – Nurse Vergel is newly assigned in the Pediatric Ward in San Isidro Medical Center. Since he will
be taking care of pediatric patients, there is a need for Nurse Vergel to be abreast with the legal aspects of
pediatric nursing care.

66. Which of the following regulates the practice of the nursing profession?
A. Department of Health
B. Professional Regulation Commission
C. Civil Service Commission
D. Professional Regulation Board of Nursing

67. One of the Vergel’s patients to undergo invasive procedure. Based on the Patient’s Bill of Rights,
sequence the persons from whom Nurse Vergel could obtain consent?

1. Parents
2. Legal guardian
3. Next of kin
4. Physician
A. 1, 2, 4, & 3
B. 1, 4, 2, & 3
C. 1, 2, 3, & 4
D. 1, 3, 2, & 4

68. Based on the Right to Privacy and Confidentiality under the Patient’s Bill of Rights, the patient has the
right to demand the following, but NOT ______ pertaining to his care as confidential.
A. Information
B. Communication
C. Records
D. financial Status

69. The Right to Information does not include ____.


A. Any change in the plan of care before the change is made
B. Extent to which payment maybe expect from Philhealth
C. Discount for the professional fee of the attending doctor(s)
D. Result of the evaluation of the nature and extent of his/her disease

70. A patient has the right to leave the hospital regardless of his physical condition provided s/he is
informed of which of the following? Select all that apply.

1. Is informed of the medical consequences of his/ her decision.


2. Releases those involved in his/her care from any obligation relative to the consequences of his decision.
3. His/her decision will not prejudice public health and safety
4. Appropriate arrangement has been made to settle the unpaid bill.
A. 1, & 2
NURSING PRACTICE II: Care of Healthy/At Risk Mother and Child
November 17 & 18, 2018
B. 1, 2, & 4
C. 3 & 4
D. 1, 2, 3, & 4

Situation – Liza is a new nurse in the Obstetrics Ward. She remember her Clinical instruction saying that a
nurse must learn how to communicate well with her patients.

71. Liza learned active listening. How will Liza demonstrate this on her patient named Grace? She Should
________.

1. Listen to what Grace verbalizes and observe how she expresses her feelings
2. Demonstrate interest on what Grace is talking about by avoiding her eyes.
3. Maintain a close body posture such as keeping arms crossed and clenching fist.
4. Tune out other thoughts and refrain from interrupting Grace while she is talking
A. 1 and 2
B. 1 and 4
C. 3 and 4
D. 2 and 3

72. Which of the following should Not be considered by the nurse in interpreting nonverbal messages of a
patient?
A. Culture, educational attainment and position in society.
B. Congruency of nonverbal expressions with spoken words.
C. Facial expression, posture, tone of voice and age.
D. Eye contact with all patients at all times regardless of race.

73. Demonstrating what she learned from school in terms of obtaining feedback, Liza’s APPROPRIATE
response to Grace’s complaint, “My breasts are engorged and it is so painful, “ should be _______.
A. “Tell me about the pain. Is it tolerable, moderate or severe?”
B. “It is alright all breastfeeding mother feel the same as you”.
C. “That’s a sacrifice that mothers should do for her babies”.
D. “Just continue breastfeeding, it will relieve you from your pain”.

74. Liza also recalled her learning on how to REFOCUS conversations. Thus, when Grace’s subsequent
messages were, “I have this painful engorged breasts and my mother-in-law wants me to bottle feed
instead of breastfeed”. What will be Liza’s BEST answer using refocusing technique?
A. “What were you saying about your mother-in-law?”
B. “All mother s-in-law are just as helpful, so it will be better to just accept her”.
C. “I think we were talking about breast engorgement which brought about.”
D. “What did you do about the pain you felt on your breast?”

75. In the course of Liza’s duty in the OB Ward, she also learned that she has to adjust her style of
communication for different types of patients. Thus, Liza should adjust to which of the following
characteristics of Grace? Select all that apply.

1. Level of usual communication


NURSING PRACTICE II: Care of Healthy/At Risk Mother and Child
November 17 & 18, 2018
2. Pace of interaction
3. Display of emotions
4. Full development of topic
A. 1, 2, 3, 4
B. 1 and 2
C. 3 and 4
D. 1, 2, 3

Situation – Nurse Kayla is assigned at the postpartum ward. The following questions refer to postpartum
patients.

76. Amy, a multiparous patient, 28 hours after Ceasarian delivery (CS), who is breastfeeding, complains of
severe abdominal cramps. Nurse Kayla explains that these are caused by which of the following?
A. Flatulence accumulation after CS.
B. Release of Oxytocin during the breastfeeding session.
C. Healing of the abdominal incision after CS.
D. Side effects of the medications Administered after delivery.

77. Three hours postpartum, a primiparous patient’s fundus is firm and midline. On perineal inspection by
Nurse Kayla, she observes a small constant trickle of blood. What will the nurse suspect for the patient to
have?
A. Uterine inversion
B. Perineal lacerations
C. Retained placenta
D. Bladder distention

78. On the first postpartum day, a primiparous patient complains of perineal pain that was unrelieved by
Ibuprofen 400 mg given two hours ago. Nurse Kayla should assess for which of the following?
A. Vaginal lacerations.
B. Perineal Hematoma.
C. History of drug abuse.
D. Puerperal infection.

79. Twelve hours after vaginal delivery, Nurse Kayla palpated the fundus of a primiparous patient and finds
it to be firm, above the umbilicus and deviated to the right. What is the Best thing for Nurse Kayla to do for
the patient?
A. Contact the physician for an order of methylergonovine.
B. Gently massage the fundus to expel the clots.
C. Encourage patient to ambulate and to void.
D. Document this as a normal finding in the patient’s records.

80. After instructing a primiparous patient about episiotomy care, which of the following indicates
successful teaching?
A. “I will wipe the area from front to back using a blotting motion.”
B. “I will put ice packs on the site 15 minutes once a day for 3 to 4 days.”
C. “Before bedtime, I will do a cold water sitz bath.”
NURSING PRACTICE II: Care of Healthy/At Risk Mother and Child
November 17 & 18, 2018
D. “I’ll use sudsy water to clean the episiotomy.”

Situation – Preterm newborns are considered high risk because of their immaturity to adapt to the
extrauterine life. They need special treatment and specialized process of nursing care.

81. The goal of a neutral thermal environment is to assist the newborns to stabilize its temperature that
does not drop below which of the following?
A. 98.6 degree F
B. 99.5 degree F
C. 97.7 degree F
D. 96.7 degree F

82. Nurses should be alert when caring for a preterm newborn, particularly for which of the following
signs?
A. Hypercalcemia
B. Premature closure of ductus arterious
C. Meconium aspiration syndrome
D. Hypoglycemia

83. The nurse starts giving information about preterm newborns to the parent. She tells them that preterm
newborns exhibits which of the following characteristics/signs? *
A. Allow elbows to be brought past the midline of the chest.
B. Exhibit an abundance of brown fat.
C. Resists when the right heel is brought to the right ear.
D. Assume significant flexed posture.

84. Hypothermia is common in preterm newborns because of their inability to control heat. Which of the
following is an EXCEPTION to the APPROPRIATE nursing intervention to prevent heat loss?
A. Using mechanical pressure
B. Drying and wrapping the baby
C. Placing the crib beside the wall
D. Doing kangaroo care

85. Which of the following conditions is NOT INCLUDED among the effects of cold stress?
A. Hypoglycemia
B. Metabolic acidosis
C. Increase intracranial pressure
D. Cerebral palsy

Situation – Nurse Melissa has been assigned in the Pediatric Ward for almost a year. Her Headnurse is on
Maternity Leave for the next 6 months. She was appointed by the Area Nurse Supervisor to be the Charge
Nurse.

86. Nurse Melissa recalls that one of the major responsibilities of a Headnurse is record keeping. These are
many reasons for keeping records in health care, but two stand out above all others. These are to ______.
NURSING PRACTICE II: Care of Healthy/At Risk Mother and Child
November 17 & 18, 2018
1. Compile a complete record of the patient’s journey through services
2. Enable continuity of care for the patient both within and between services.
3. Make available for legal purposes
4. Protect nurses from ligitation
A. 1 & 4
B. 2 & 3
C. 3 & 4
D. 1 & 2

87. The following statements demonstrate to Nurse Melissa that record keeping is good, EXCEPT:
A. Entries are dated and timed as close to the actual time of the events as possible.
B. Relevant information about the condition of the patient/ client at any given time and the measures you
have taken to respond to their needs.
C. Abbreviation is used.
D. A full account of your assessment and the care you have planned and provided.

88. On record keeping, the Area Supervisor reminds Nurse Melissa that for most Nurses who find
themselves involved in legal proceedings, the best defense to win the case is _____.
A. Honesty
B. Good memory
C. Quality of nursing documentation
D. Knowledge

89. Which statement is incorrect regarding record keeping?


A. Patients now have a legal right to see their own records.
B. Keeping records of patient care and treatment is NOT an integral part of nursing practice.
C. Any later alteration or addition must be carefully indicated, insuring that the original entry can still be
clearly read.
D. All records must be signed by the person making the entry together with the date and time of the entry.

90. When the patients asks where to get his/her medical record after discharge, the correct answer is
_______.
A. Office of the Attending Physician
B. Medical Record Section
C. Admission Section
D. Office of the Administrative Officer

Situation – Nurse Myrna, a nurse in Barangay Mayamot Health Center, will conduct Health education to the
residents of the Barangay on Reproductive Health.

91. As a Community Nurse, Nurse Myrna should be able to demonstrate knowledge of current laws on
reproductive health in which she practices. One of the topic topics that Nurse Myrna will discuss is RA
10354 which is entitled ________.
A. Reproductive Health Act of 2012
B. The Responsible Parenthood and Reproductive Health Act of 2012
C. The Responsible Parenthood and Reproductive Parenthood of 2012
NURSING PRACTICE II: Care of Healthy/At Risk Mother and Child
November 17 & 18, 2018
D. RH Bill

92. This Law provides reproductive health care program to all citizens prioritizing, which of the following?

1. Women
2. The poor
3. Marginalized
4. Those in vulnerable or crisis situations
A. 1, 2, 3 and 4
B. 1, 2 & 3
C. 1 only
D. 1 & 2

93. When conducting health education on Reproduction Health, it is important that Nurse Myrna examines
her values because of which of the following reasons?

1. We all hold personal values that can influence how we respond to our clients.
2. One’s values may change in response to life experiences.
3. Encounters with clients and colleagues may influence your beliefs without your having much of a chance
to reflect on these changes.
4. Others exist at a deeper level, so that we don’t necessarily recognize the influence they have on our
behavior and judgments as health care providers.
A. 1, 2 and 3
B. 1, 2, 3 and 4
C. 1, 3 and 4
D. 2, 3 & 4

94. Reproductive Health service include the following, EXCEPT:


A. Gender-based Violence
B. Finding the correct partner
C. Management and prevention of maternal and perinatal
D. Malnutrition and anemia

95. Factors that affect reproductive health care, which of the following?

1. Economic circumstances,
2. Education,
3. Living conditions and family environment,
4. Social and gender relationships
A. 1 & 2
B. 1, 2 and 3
C. 1, 2, 3, & 4
D. 2, 3, & 4
NURSING PRACTICE II: Care of Healthy/At Risk Mother and Child
November 17 & 18, 2018
Situation – Nurse Miriam Kate is the Charge of the Obstetrics & Gynecology Ward of a government hospital.
Nurse Angelina is one of the staff nurses in that ward. The Ward is beset by problems such as having
inadequate supplies.

96. What is the BEST initiative that Nurse Angelina can do, should there be non-availability of supply of
alcohol used for hand hygiene that is vital in preventing cross-infection?
A. Requisition of the items from Central Supply before the supply runs out.
B. Inform the Head nurse of its non-availability.
C. Have your own alcohol until hospital supply is available.
D. Request the patients to provide their own alcohol.

97. One major way of preventing cross-infection is frequent handwashing. However, today, Nurse Angelina
just found out there is water interruption. The INITIAL step that Nurse Angelina ought to do is report the
problem to ____.
A. Head nurse
B. Manila Water
C. Chief Nurse
D. Nurse Supervisor

98. Nurse Angelina as a beginning assigned on the Obstetric Ward was asked by Head nurse Mimi an
example of how she will be able to manage her environment. Which of her responses are CORRECT? Select
all that apply.

1. Check availability and expiry date of fire extinguishers in the Obstetric Ward.
2. Compliance of Republic Act 9003 – ecological solid waste management
3. Provision of a separate container for each type of waste for on-site collection properly marked as
“compostable”, “non-recyclable”, “recyclable” or special waste”.
4. Proper isolation of patients known or suspected of harboring infectious diseases.
A. 1, 2 & 4
B. 1, 3 & 4
C. 1, 2, 3 & 4
D. 2, 3 & 4

99. Should Nurse Angelina request for Alcohol-based hand rub to be effective for application to the hands
for reducing the number of viable microorganism on the hands, the ethyl content should be _____.
A. 60
B. 90
C. 70
D. 95

100. Nurse Angelina discovered that bed mattresses in the Pay Ward slide easily, either to the left or right
side of the bed causing patient fall. What would be Nurse Angelina’s MOST appropriate way to provide
patient’s safety?
A. Use restraint on Patient.
B. Raise upper and preferably lower fall rails at all times.
C. Frequent monitoring of patient.
NURSING PRACTICE II: Care of Healthy/At Risk Mother and Child
November 17 & 18, 2018
D. Ensure 24-hour responsible watcher.

ANSWER KEY FOR NURSING PRACTICE II

1.D 11.A 21.A 31.C 41.A 51.C 61.D 71.B 81.C 91.B
2.C 12.D 22.C 32.B 42.A 52.D 62.C 72.D 82.D 92.A
3.C 13.D 23.C 33.B 43.D 53.C 63.B 73.A 83.A 93.B
4.B 14.B 24.A 34.D 44.B 54.A 64.D 74.D 84.C 94.B
5.A 15.B 25.C 35.D 45.A 55.A 65.C 75.B 85.D 95.C
6.C 16.B 26.A 36.A 46.B 56.C 66.D 76.B 86.C 96.B
7.B 17.B 27.D 37.A 47.C 57.A 67.D 77.B 87.C 97.A
8.B 18.C 28.B 38.B 48.C 58.B 68.D 78.B 88.C 98.C
9.D 19.A 29.C 39.A 49.D 59.C 69.C 79.C 89.B 99.A
10.D 20.D 30.A 40.C 50.D 60.B 70.D 80.A 90.B 100.B
NURSING PRACTICE III: Care of Clients with Physiologic and Psychosocial Alterations, Part A
November 17 & 18, 2018
Situation – Ms. Limjap is the staff development coordinator of a Tertiary Hospital and has an In-service
training program to a group of newly hired nurses for the month. Due to the many patients they have in the
hospital afflicted with Intestinal and rectal disorders, a refresher session on these cases became the focus
of their discussion.

1. The first question she asked from Nurses Gayle is, how do you differentiate a patient with Crohn’s
Disease from that of ulcerative colitis in terms of the following:
A. Presence of bleeding: severe in Crohn’s Disease while in Ulcerative Colitis bleeding is mild.
B. Presence of diarrhea: severe in Crohn’s Disease in Ulcerative Colitis it is mild.
C. Affected area: Crohn’s Disease is the descending while in Ulcerative Colitis is the ascending colon
D. Course of the disease: Crohn’s Disease is prolonged and variable. Ulcerative Colitis has a remission and
exacerbation

2. Nurse Limjap further asked another orientee, Ms. Sharon “In preventing recurrence of Crohn’s and
Ulcerative Colitis, what specific therapeutic regimen is effective to both condition? The BEST response of
nurse Sharon is.
A. Cortecosteroids
B. Anti-cholinergic
C. Antacids
D. Anti-spasmodics

3. As a nurse, Ms. Lyn, what systematic complication do you likely expect when you are caring for a patient
with Crohn’s Disease? The BEST response of Nurse Lyn to Ms. Limjap is.
A. Megacolon
B. Perforation
C. Small bowel Obstruction
D. Hemorrhage

4. The succeeding questions of Ms. Limjap to the orientees include the following: What diagnostic test is
usually ordered to patients with Ulcerative Colitis in order to distinguish this condition from other diseases
of the colon?
A. Proctosigmoidoscopy
B. Albumin Study
C. Colonoscopy
D. Ultra Sound

5. If you are carrying for patients who are passing out large volume of stools brought about by Ulcerative
Colitis, which of the following nursing diagnoses is the MOST appropriate to be formulated and
incorporated in your nursing care plan?

1. Imbalanced nutrition less than body requirements R/T impaired absorption


2. Rick for Deficient Fluid Volume R/T abnormal fluid loss
3. Risk for Ineffective tissue perfusion R/T low hemoglobin
4. Acute Diarrhea R/T inflammation of the bowel
A. 2 & 3
B. 1, 2 & 3
NURSING PRACTICE III: Care of Clients with Physiologic and Psychosocial Alterations, Part A
November 17 & 18, 2018
C. 1, 2, 3 & 4
D. 1 & 2

Situation – Ms. Betsy, 56, year, a former dressmaker, married was brought the medical counter because of
severe abdominal pain radiating to the scapular region. Health history revealed that she has been fond of
eating fatty prepare food. Patient appeared jaundice with facial grimaces. The physician on duty (POD)
examined her and an impression of Cholelithiasis was given by the POD. Patient was ordered to be
admitted for further work-up and possible surgery.

6. During the admission interview, which of the following questions would the Nurse APPROPRIATELY asks
when a patient is complaining of biliary colic?
A. When do you feel the pain? Is it before you sleep at night?
B. Do you feel the pain and tenderness in the left upper quadrant of your abdomen?
C. Does the pain last for thirty minutes or less?
D. When do you feel the pain, is it after a heavy meal?

7. During the comprehensive assessment of the Nurse, which of the following would the Nurse likely to find
when Ms. Betsay has an obstructive jaundice?
A. Itchiness of the abdomen
B. Clay-colored stool
C. Reddish-colored urine and stool
D. Presence of urobilinogen in the urine

8. The POD ordered the patient for bile duct exploration with open cholecystectomy as the Xray and
diagnostic tests revealed that she has multiple stones obstructing the biliary tract. After surgery a T-tube
was connected as a drain. The nurse on duty (NOD) found out after twelve hours (12) that there was a
remarkable decrease from 250ml to 90 ml in the amount of bile drainage from the T-tube. What is the
PRIORITY nursing action should the NOD do?
A. Report the incident to the physician on duty.
B. Assess the tube for any obstruction
C. Irrigate the tube with the normal saline solution
D. Change the position of the patient.

9. After surgery, Ms. Betsy should be watched for ONE of the MOST common complications that may arise.
The condition is
A. Pleurisy
B. Emphysema
C. Broncho Pneumonia
D. Pneumonia

10. The Nurse on duty is preparing for discharge plan for Ms. Betsay. What dietary restriction should she
emphasize so that recurrence of stone formation will be prevented?
A. Calamansi Juice, white bread and chicken breast salad.
B. Whole milk, butter, cheese and fried meat.
C. Steamed Fish, broiled vegetable and lean meat
D. Broiled banana, vegetable salad and white meat of chicken.
NURSING PRACTICE III: Care of Clients with Physiologic and Psychosocial Alterations, Part A
November 17 & 18, 2018

Situation – Mister Gleen is a Senior High School student who has recently been diagnosed to have bronchial
Asthma. Eating seafoods and exposure to animals usually participate his attack. One day he was rushed to
the Emergency Room (ER) because of asthmatic attack. He was given Theophylline and Cromolyn sodium
for his asthma. You are the nurse-on-duty in the emergency room.

11. When admitting Master Gleen your assessment findings will LIKELY be the following, EXCEPT:
A. Increased tactile fremitus
B. Resonant to hyper-resonance
C. Wheezing sounds
D. Crackles

12. When administering theophylline to the patient, what PRIORITY drug-induced adverse reaction should
you monitor as a nurse?
A. Constipation
B. Bradycardia
C. Diarrhea
D. Restlessness

13. When evaluating the effects of the drug (Theophylline), what is the positive outcome do you observe?
Decreased in?
A. Breath Sounds
B. Wheezing
C. Body Temperature
D. Allergic Reaction

14. One day, Master Gleen asked his medication Nurse, why he has to take theophylline. The BEST response
of the nurse is it:
A. “dilates the bronchial tree and facilitate breathing.”
B. “produces anti-histamine effect and prevent allergic reaction.”
C. “minimize the occurrence of seizure.”
D. “constricts the bronchial tree and decreases inflammation.”

15. What nutritional instruction should you advise Master Gleen when taking Theophylline? He should
AVOID taking?
A. Milk and milk products
B. Fruit and vegetable juices.
C. Soda and Non-carbonated drinks
D. Hot tea or coffee.

Situation – Belen, 42 years old, who is diabetic is brought to the Emergency room because her husband
observed her to be disoriented for the past two days. Her blood sugar is 300mg/dl. She has fruity breath
odor and occasional arrythmia. The physician suspected that the patient is having Diabetic Ketoacidosis
(DKA). You assess the patient.

16. What do you expect the physician to order considering the clinical manifestations of Ms. Belen?
NURSING PRACTICE III: Care of Clients with Physiologic and Psychosocial Alterations, Part A
November 17 & 18, 2018
A. 5% d/nss
B. 0.9 % NSS
C. Ringers Lactate
D. 5% D/W

17. As a nurse, you understand that the Type 2 Diabetes is characterized by the following, EXCEPT:
A. Onset is acute
B. Insidous in onset
C. Decrease in the number and size of Beta Cells
D. Insulin resistance with Insulin deficiency

18. Ms. Belen started to develop autonomic neuropathy, a common complication of diabetes mellitus. You
are aware that this condition can lead to what condition?
A. Gastro-intestinal and urinary incontinence
B. Development of deep ulcer formation
C. Decreasing vision when ambulating
D. Tiredness and easy fatigability

19. While in the hospital, the physician in coordination with the nurses, dietitian and other health
professionals has to consider total patient care. What particular meal plan will be carried out by the nurse
for Ms. Belen? Food has to be

1. 60 to 70% caloric intake from CHO and monosaturated fats


2. Prepared from lower monosaturated oil like olive oil
3. Prepared from avocado oil and nuts
4. High in CHO, high in saturated fats and low protein
A. 1, 2 & 3
B. 2 & 3
C. 1, 2, 3 & 4
D. 1 & 2

20. Ms. Belen will be discharged in a few days time. What should be include in your care plan as part of
your foot infection prevention?
A. massage feet and legs with mild lotion
B. Use tight stockings when ambulating
C. Trim toe nails with ordinary nail cutter
D. Use heating pad whenever patients feels cold

Situation – Rafael is the Nurse in Charge of the Central Supply Room (CSR) in Hospital Kundiman. A 100 bed
capacity, she knows that she has to distribute supplies to the different wards in a manner that is equitable
and just.

21. What should be her FIRST strategy in order to meet her objective? To collect data on
A. The weekly wastages of supplies
B. Months where consumptions are high
C. The daily consumption of supplies
NURSING PRACTICE III: Care of Clients with Physiologic and Psychosocial Alterations, Part A
November 17 & 18, 2018
D. Frequency of events like disasters

22. Time is an important resource in managing supplies. Hence, it is BEST for the nurse who manages the
CSR to requisition supplies and materials _____ days prior to their consumption. Which is this?
A. 40
B. 30
C. 50
D. 60

23. During rainy days, there are more 0-6 years old pediatric patients who are hospitalized due to
respiratory and gastrointestinal diseases. Hence, there are more who would need intravenous therapy,
what size of cannula must be ordered by the nurse? Gauge.
A. 24-26
B. 18-22
C. 16-18
D. 22-24

24. Which of the following is the correct principle of stocking intravenous fluids in CSR?
A. A mix of previous and present stocks
B. Present stock first
C. Separation of the present from previous stocks
D. Previous stock first

25. In order to find out oxygen wastes. Nurse Rafaela must advise the ward nurse to keep track of it on
which of the following time frame? At Least every?
A. Shifts
B. Day
C. Weak
D. Hour

26. Safety and quality care are considered priority by the surgical team before, during and after surgery.
The quality outcome indicators for surgery that is ALARMING to the team includes which of the following?
A. Patients with appropriate specific operative hair removal completed.
B. Prophylactic antibiotic discontinued within 24 hours after surgery.
C. Indwelling catheter removed within 24 hours post-operatively
D. Cardiac surgery patients uncontrolled 6 AM blood glucose post-operatively.

27. One of the Risk Factors for patients going for surgery is nicotine use. If patients are cigarette smokers,
which of the following Nursing actions has to be implemented by the nurse post-operatively?

1. Encourage breathing, coughing and early ambulation.


2. Offer fluid intake 2500-3000 ml per day, if not contraindicated.
3. Apply nicotine patch to reduce withdrawal symptom, if ordered.
4. Support efforts to quit smoking through smoking cessation program.
A. 2 & 3
B. 1, 2, 3, & 4
NURSING PRACTICE III: Care of Clients with Physiologic and Psychosocial Alterations, Part A
November 17 & 18, 2018
C. 1 & 2
D. 1, 2, & 3 28.

28. When a patient is for surgery, preoperative medications are administered to achieve specific
therapeutic outcomes. Which of the following are examples of ANTICHOLINERGIC drugs which are
administered to reduce oral and respiratory Secretions?

1. Atropine sulfate
2. Glycopyrrolato
3. Scopolamine
4. Ranitidine
A. 1, 2, 3 & 4
B. 1, 2 & 3
C. 2 & 3
D. 1 & 2

29. Anesthesia is used to produce sedation reflex loss and muscle relaxation during a surgical procedure.
When a patient is for surgery of the lower abdomen, perinuem and lower extremities, the surgeon will
likely order what TYPE of anesthesia?
A. General
B. Epidural
C. Conscious
D. Spinal

30. Based on the World Health Organization surgical safety checklist, which of the following is NOT in the
sign-out area of the checklist?
A. Surgeon, anesthesia professional and nurse verbally confirm patient, site and procedure to be done.
B. Nurse confirms with the team instruments, sponge and needle counts are correct.
C. Whether there are equipment problem to be addressed.
D: Nurse confirms, with the team name of procedure recorded and how specimen is labeled.

Situation - Ethical decision making in clinical health care is becoming more complex, this is brought about
by the advancement of science and technology, cultural beliefs and patient's rights.

31. When a patient refuses to undergo insertion of a pacemaker despite the presence of a third degree AV
block and respected by the health team is what bioethical principle?
A. Beneficence
B. Autonomy
C. Nonmaleficence
D. Justice

32. The health care professionals consider the following as ethical dilemma EXCEPT which ONE of the
following. When the _____.
A. answer will produce profound effect on the situation
B. choice is made between equally unfavorable alternatives on basis of risks and benefits
C. situation involves a conflict between two moral imperatives
NURSING PRACTICE III: Care of Clients with Physiologic and Psychosocial Alterations, Part A
November 17 & 18, 2018
D. situation cannot be solved by a review of scientific data

33. Nurse Blanca brought her patient to the chapel by wheelchair as promised after checking the blood
pressure of another patient in the ward, is an example of what ethical principle?
A. Veracity
B. Justice
C. Beneficence
D. Fidelity

34. A newly hired graduate nurse Belen is on the second week of orientation by the In-service coordinator.
Two of the nurses in the Intensive Care Unit (ICU) called "ON sick" so Ms. Belen was pulled out by the
Supervisor to report to the ICU as she could not get a replacement due to staffing shortage. This ethical
dilemma in management is an example of ______.
A. professional ethics
B. governance ethics
C. organizational ethics
D. clinical ethics

35. Ms. Simon who is assigned in the Emergency room was attending to an older woman who was having
severe abdominal pain with a very unstable blood pressure. The doctor ordered a very high dose of
morphine So4 which Nurse Simon questioned. What ethical principle is applied in this situation?
A. Veracity
B. Nonmaleficonce
C. Justice
D. Beneficence

Situation - Quality documentation is an essential component of the nurse’s role in the Health facilities.
Nurse Gladys is assigned in the Female medical ward where most of her patients have cardiac and vesicular
disorders.

36. Nurse Gladys is conducting an assessment of Mrs. Carpio who has edema of the lower extremities due
to congestive heart failure. If edema is graded on a scale of 1 (no visible edema in the leg) to 4+ (Leg very
swollen), of what the will Nurse Gladys document in her chart if her assessment findings of the edeme is
slightly swollen?
A. +0
B. +1
C. +2
D. +3

37. What specific term should Nurse Gladys write in her charting when a patient is suffering from a change
in the angle the nail base greater than 180 degrees due to congenital heart disease?
A. Peripheral neuropathy
B. Peripheral cyanosis
C. Inflammation of the fingers
D. Clubbing of the fingers
NURSING PRACTICE III: Care of Clients with Physiologic and Psychosocial Alterations, Part A
November 17 & 18, 2018
38. Mrs. Carpio has an order of Thiaxice diuretics for her edema. The following is the entry that you expect
to be written in her progress report EXCEPT _____.
A. serum electrolyte monitored everyday
B. weight taken before drug is administered
C. intake and output done every shift
D. drug is administered at 8 o'clock in the evening

39. Which of the following is NOT APPROPRIATE description and written in the Nurses notes when a patient
has a pitting edema?
A. Degree of edema
B. Degree of temperature
C. Duration of indentation
D. Depth of edema

40. M. Ruby 20 year old also a patient of Nurse Gladys is suffering from Rheumatic fever due to a previous
streptococcal infection. In monitoring patient's status, the nurse should document which of the following
TYPICAL manifestations EXCEPT:
A. increase body temperature
B. non-pruritic rashes
C. voluntary muscle spasm
D. migratory joint pains

Situation - Mrs. Cruz, 68 years old, 5 2" tall, weighs 80 lbs. came to the General hospital Because of
diarrhea and eating problems. Her BMI is <18.5 and has not eaten anything for almost 12 hours.

41. Which of the following is the APPROPRIATE initial question to be asked to the patient upon admission?
A. “Does stress give you problem with your eating habits?"
B. “Do you recall the food you have taken prior to your admission?"
C. “Do your family members allot enough budget for your nutritional needs?"
D. “What is your usual eating pattern?"

42. Considering the clinical manifestations presented by the patient, how BEST should the nurse state her
nursing diagnosis?
A. Imbalanced nutrition less than body requirements R/T poor absorption of food.
B. Activity intolerance R/T inadequate caloric intake.
C. Risk for infection R/T poor nutritional status
D. Self-care deficit R/T decreased strength.

43. The doctor ordered a nasogastric tube (NGT) inserted before administering Osmolite tube feeding.
Which of the following nursing measures should the nurse do?
A. Assess the presence of bowel sounds.
B. Evaluate if patient's residual is <50 ml
C. Warm the Osmolite to 98.8 F.
D. Flush the NGT with 10 ml of water.
NURSING PRACTICE III: Care of Clients with Physiologic and Psychosocial Alterations, Part A
November 17 & 18, 2018
44. Which of the following results will describe a normal physical finding when performing gastro-intestinal
assessment?
A. Eversion of umbilicus upon inspection.
B. Tympany on percussion of the abdomen.
C. Gurgling abdominal sounds upon auscultation
D. Liver edge of 1-2cm below the right costal margin on palpation.

63. In the process of doing performance appraisal of nursing staff, Nurse Karen is guided by the basic goals
of comprehensive appraisal system. Which of the following is included in this process?

1. Determine the capability required for the job


2. Match abilities of the nurse with the job requirements
3. Improve nurse's capability
4. Enhance nurse’s motivation
A. 1, 2, and 3
B. 3 and 4
C. 1 and 2
D. 1, 2, 3 and 4

64. In conducting performance appraisal, Nurse Karen as a manager, is aware that bias can happen when
assessing her staff with just one method. To minimize subjectivity multiple methods can be employed.
Which of the following methods is MOST applicable for a staff position level?
A. peer review
B. Management by objectives
C. 360 degree evaluation
D. self-evaluation

65. Nurse Karen has a staff nurse who has been with her unit for almost two years. Recently, she has
observed that her work performance is getting substandard, such that she misses deadlines and is
becoming uncooperative. What is BEST approach to be done to this problem?
A. Mentoring
B. Scolding
C. Counseling
D. Coaching

Situation - A domain in the development of the competency of the practicing nurse is ones' Legal
responsibility.

66. The graduates from the colleges of Nursing are being prepared to take the Nursing Licensure
Examination (NLE) with the main goal to:
A. provide opportunity for overseas employment
B. determine the standards of nursing practice
C. protect the public from incompetent nurse practitioner
D. limit the practice of the profession
NURSING PRACTICE III: Care of Clients with Physiologic and Psychosocial Alterations, Part A
November 17 & 18, 2018
67. Ms. Libby is a professional nurse who was approached by her cousin to be in a Television show where
she has to wear the nurses uniform and advertise a commercially prepared beauty product for use in their
work setting. Her mother is presently having dialysis so the accepted the offer. Ms. Libby is violating the
_______.
A. nursing law
B. nursing code of ethics
C. nursing jurisprudence
D. consumers fraud law

68. Mr. Larson is scheduled for Coronary Artery-by-pass graft (CABG) surgery. He has to sign an informed
consent prior to operation. The validity of this consent is how long?
A. 36 hours after surgery
B. 24 hours after surgery
C. after the surgical procedure
D. 12 hours after surgery

69. A patient was admitted in a medical center by his physician for an Exploratory laparotomy. After
discharge for two days, he was complaining of chills and fever. The abdominal incision was passing out
yellowish purulent discharge. He had an abdominal Xray and was was found out to have 3 pcs of gauze
inside the abdomen. A libel suit was filed by the family. Who is liable in this case?

1. Surgeon
2. Circulating nurse
3 Anesthesiologist
4. Hospital
A. 1, 2, 3, & 4
B. 1, 2, & 3
C. 1 & 2
D. 2 & 3

70. Nurse Rose is assigned to a patient in ICU pronouned who is dying due to cardiac arrest. After two
hours, the partient was pronouned dead. Which of following health status should guide the nurse is the
patient is declared dead?
A. All body systems are no longer functioning
B. Flat sign of electro-cardiogram
C. Was no more pain sensation
D. Stoppage of heart beat and blood pressure
NURSING PRACTICE III: Care of Clients with Physiologic and Psychosocial Alterations, Part A
November 17 & 18, 2018

KEY ANSWER FOR NURSING PRACTICE III

1.D 11.A 21.C 31.B 41.B


2.B 12.D 22.B 32.D 42.B
3.C 13.B 23.A 33.D 43.A 63.D
4.C 14.A 24.D 34.C 44.B 64.C
5.B 15.D 25.A 35.B 65.C
6.D 16.A 26.D 36.C 66.B
7.B 17.A 27.B 37.D 67.B
8.B 18.A 28.C 38.D 68.B
9.D 19.D 29.B 39.B 69.A
10.B 20.A 30.A 40.C 70.A
NURSING PRACTICE III: Care of Clients with Physiologic and Psychosocial Alterations, Part A
November 17 & 18, 2018
NURSING PRACTICE IV: Care of Clients with Physiologic and Psychosocial Alterations, Part B
November 17 & 18, 2018
Situation - A landslide had occurred in x province due to persistent and heavy rainfall. According to
information received by the Emergency Department Staff, there were about 150 people injured.

1. The nurse prepares the department to receive the injured. This situation would be BEST classified as
a/an:
A. Unnatural Calamity
B. Natural Calamity
C. Mass Casualty Incident
D. Accidental Disaster

2. The first responders to the scene of disaster would MOST likely use which of the following tools to
separate victims for easy recognition of those in need of immediate care.
A. Number Codes
B. Triage Tag
C. Category Tags
D. SALT Triage

3. The ED nurse plans the coding for disaster victims. Which colors are MOST associated with triage?

1. Green
2. Yellow
3. Black
4. Blue
5. White
A. 2, 3, 4, 5
B. 1, 2, 3, 4, 5
C. 1, 2, 3, 4
D. 1, 2, 3

4. The ED nurse receives a victim with severe life- threatening injuries and most likely will not survive. The
victim would be triaged as:
A. Yellow
B. Red
C. Black
D. White

5. One of the victims, a female was brought to the ED crying hysterically and looking frightened. She
exclaims she was inside her house when the landslide occurred and just barely escaped. What is the BEST
action by the nurse?
A. Refer her to the proper authorities for counselling.
B. Triage her and give appropriate treatment.
C. Call a physician to talk to her.
D. Advise her to go home with a relative.

Situation - A nine year old male child is hospitalized for burns on the right arm, right leg and abdomen. The
nurse documents the treatment performed on the child.
NURSING PRACTICE IV: Care of Clients with Physiologic and Psychosocial Alterations, Part B
November 17 & 18, 2018

6. The nurse determines the extent of burns using the rule of nines. Which of the following assessment
findings should the nurse document?
A. 18% of the child’s body surface is burned.
B. 45% of the child’s body surface is burned.
C. 50% of the child’s body surface is burned.
D. 25% of the child’s body surface is burned.

7. The nurse writes a nursing diagnosis for the child which is the basis of care for the first 24 hours of
admission. The MOST appropriate nursing diagnosis would be _______:
A. Fear and Anxiety
B. Disturned Body Image
C. Risk for Infection
D. Impaired Mobility

8. The physician writes an order for the client. Infuse D5 water 500 cc to run for 8 hours. The IV micro set
delivers 60 drops per ml. How many drops should the nurse regulate the flow and record it in the client’s
chart?
A. 50 drops per minute
B. 62 drops per minute
C. 35 drops per minute
D. 30 drops per minute

9. The nurse commits an error in documenting the care of the burnt child. She consults the charge nurse to
find out if the hospital has an established policy on correcting documentation errors. Which of the
following is an accepted form for correcting errors?
A. Enclose in parenthesis the erroneous statements, draw a line across the statement, and make the
correct entry above the 1 drawn.
B. Correct the error by applying correction fluid or tape and write the correct entry over it.
C. Cross through the erroneous word or statement with a double line, affix your initials, write the pharse
“mistaken entry” then write the correct information.
D. Use the slide rule method. Cross through the erroneous word or statement with a single line, affix your
initials, write the date and time the correction was made, the write the correct information.

10. The nurse is aware that documentation requires the following EXCEPT: Documentation should:
A. Be systematic and organized
B. Comply with policy standards of the health care facility.
C. Present exact and correct details pertinent to the event.
D. Include reactions and interpretations of the nurse on the event.

Situation - The assistant chief nurse of X hospital attends a seminar on quality and performance
improvement. The seminar is to increase awareness on how performance improvement affects client care
and the health care organization.

11. Which of the following principles of quality improvement is MOST appropriate for patient care?
A. The priority is to benefit clients and all other internal and external customers.
NURSING PRACTICE IV: Care of Clients with Physiologic and Psychosocial Alterations, Part B
November 17 & 18, 2018
B. Improvement of the quality of service is a continuous process.
C. Improvement opportunities are developed by focusing on the work process.
D. Quality is achieved through the participation of everyone in the organization.

12. The assistant chief nurse understands that the implication of quality improvement for client care can be
measured by the overall value of care. Outcomes can be measure by which of the following?
A. Client’s recommendation
B. Clinical out come
C. Cost of care
D. Client satisfaction

13. The assistant chief nurse is oriented on the various improvement strategies at the organization level.
One of these strategies is benchmarking. Which of the following describes the bench marking process?
A. Comparing data in the literature with the data collected per client.
B. Reviewing own unit’s data for opportunities.
C. Collecting data of the individual client.
D. Comparing on data with that of other organization to identify opportunities.

14. The assistant chief nurse understands that a sentinel event review is one strategy to improve the health
care delivery system. The FIRST action to be initiated following a sentinel event is for the assistant chief
nurse to ____.
A. Conduct an immediate investigation
B. Conduct a root cause analysis
C. Recommend corrective action of personnel involved
D. Recommend what action personnel involved should avoid

15. Quality improvement can be achieved through the participation of everybody in the health care
organization at all levels. Which of the following is an example of nurses’ participation as an individual?
A. Work with others in the unit to change the way client’s report is given to be more time efficient.
B. Suggest the process for notifying the pharmacy department about a missing medication.
C. Participate in a term to find a solution.
D. Change the activities of her day to spend more time with a client’s family.

Situation - Nurse Edna, a charge nurse of an oncology unit attends a seminar on evidence based nursing
practice.

16. The conference speaker explained the difference between evidence based practice and evidence based
nursing practice. Which of the following is NOT TRUE regarding evidence based nursing practice? Evidence-
based nursing practice ____.
A. Has a medical focus
B. Uses a theory derived and research based information in making decision about nursing care delivery.
C. Considers the patient’s need and preferences based on nursing theory and research
D. Is a strategry used to improve patient care outcome.

17. The speaker encouraged the nurses to participate in the use of evidence- based nursing practice. The
speaker said that nurses must ____.
NURSING PRACTICE IV: Care of Clients with Physiologic and Psychosocial Alterations, Part B
November 17 & 18, 2018

1. Participate in the development, use, or evaluation of practice guidelines.


2. Read and analyze outcome of research studies
3. Involve themselves in everyday patient care and nursing practice.
4. Know why they are doing what they are doing
A. 2, 3 and 4
B. 1 and 2
C. 1 ,2, 3 and 4
D. 1, 3 and 4

18. Nurse Edna is aware of the importance of patient outcomes as a measure of quality care. Which of the
following statements is TRUE regarding patient-focused outcome indicators? To _______.

1. Achieve safe, quality, cost effective care for patients in daily practice.
2. Realize that individual nursing practice styles directly affect the rates at which patients recover.
3. Prevent development of unnecessary complications and injury.
4. Determine satisfaction rate from patients or family care received from nursing staff.
A. 1, 2 ,3 and 4
B. 1 and 3
C. 2 and 4
D. 1, 3 and 4

19. A model for using evidence- based practice was presented. The model has the following elements: Plan,
Do, Study and Act (PSDA). If nurse edna wants to utilize this model to improve ward management, what
questions will she ask?

1. What are we trying to accomplish?


2. How well do we know that a change is an improvement?
3. Will the patient be satisfied with the improvement?
4. What change/changes can we make that will result in an improvement?
A. 2 and 4
B. 3 and 4
C. 1 and 3
D. 1,2,3 and 4

20. Nurse Edna is aware that patient care improvement must be based on which of the following?
A. Total cost of health care
B. Building and apply knowledge
C. Hospital policy on how to staff a nursing unit.
D. Accreditation standards

Situation - The nurse cares for a client with cancer who had lung surgery. The nurse educates the client on
breathing exercise and ambulation.

21. The nurse teaches the client how to deep breath effectively after a lobectomy. The nurse instructs the
client to:
NURSING PRACTICE IV: Care of Clients with Physiologic and Psychosocial Alterations, Part B
November 17 & 18, 2018
A. Contract the abdominal muscles, take a deep breath through the mouth and exhale slowly as one trying
to blow out a candle.
B. Relax the abdominal muscle, take a slow deep breath through the nose, and hold it for 3 to 5 seconds.
C. Relax the abdominal muscle, take deep breath through the mouth and exhale slowly for 15 seconds
D. Contract the abdominal muscles, take a slow deep breath through the nose, and hold it for 3 to 5
seconds.

22. The client asks the nurse how much of his lungs are removed. The nurse responds based on information
that in lobectomy, a lobe is removed. In a wedge resection, which of the following is removed?
A. A small, localized are near the surface of the lung
B. One entire lung
C. Two lobes of the lung
D. A segment of the lung, including a bronchiole and its alveoli.

23. The client asks the nurse, what will fill the space where the lobe was? The correct response would be:
The _____.
A. Lung space will be filled up with serous fluid.
B. Surgeon filled the space with gel.
C. Space stays empty
D. Remaining lobe or lobes over expand to fill the space.

24. On the second post-operative day, the nurse auscultates the lungs and determines scattered crackles
bilaterally. Which of the following interventions would be MOST appropriate for the nurse to perform?
A. Encourage deep breathing and ambulation as soon as the client is able.
B. Encourage coughing and check the water seal system
C. Reduce the frequency of pain medications and increase the suction in the water seal bottle.
D. Perform endotracheal suctioning every shift.

25. The nurse teaches the client to perform which of the following exercises to prevent shoulder ankylosis?
A. Raise and lower the head
B. Turn from side to side
C. Raise the arm on the affected side over the head
D. Flex and extend the elbow on the affected side

Situation - Nurse Elma speciaized in emergency nursing. She collaborates with ED physician, triage team
and other members of the health team provide care to clients of all age- groups with various illnesses or
injuries coming to the emergency department.

26. When clients are wheeled into the ED, Nurse elma applies a triage priority rating system. When a client
must be treated immediately otherwise the client’s life, limb or vision are threatened, the category is ____:
A. Low urgent
B. Urgent
C. Emergent
D. Non Urgent
NURSING PRACTICE IV: Care of Clients with Physiologic and Psychosocial Alterations, Part B
November 17 & 18, 2018
27. A client with spinal cord injury due to trauma is brought to the ED by EMS personnel. Nurse Elma
assesses the client. Which of the following will the nurse suspect the client to manifest?
A. Tachycardia
B. Diaphoresis
C. Pain
D. Temporary Loss of relax function

28. The neurosurgeon performs a neurological assessment on the client. Based on the Glasgow Come Scale,
the client has a total score of 8. Nurse Elma recognizes this score to indicate which type of injury?
A. Severe injury
B. Minor Injury
C. Moderate injury
D. No injury

29. Nurse Elma writes a nursing diagnosis. Which of the following diagnoses is a PRIOPRITY nursing
diagnosis for a client with spinal cord injury?
A. Risk for infection
B. Dysreflexia
C. Ineffective airway clearance
D. Ineffective breathing pattern

30. The neurosurgeon determines the client’s spinal cord injury is at the level of T5. Nurse Elma is alerted
when the client complains of severe headache, is diaphoretic, a head and neck appear to be flushed. She
takes the vital signs. Plus rate is 47 beats per minute and blood pressure is 220/114 mmhg. Based on the
assessment data, nurse Elma concludes that the client need IMMEDIATE treatment for _____:
A. Spinal shock
B. Autonomic Dysreflexia
C. Pulmonary Embolism
D. Malignant Hypertension

Situation - Nurse Tina admits a 25 year old female for severe angioedema involving the face, hands and
feet. The patient further complains of burning and stinging of the lesions.

31. The nurse interviews the client. Which of the factors would be a MOST significant risk factor for
allergies?
A. A recent upper respiratory infection
B. Family history of allergic rections
C. Living in a third world country
D. Exposure to fungal infection

32. Based on assessment findings, nurse tina writes a nursing diagnosis is:
A. Risk for Injury
B. Altered Comfort
C. Risk for Infection
D. Impaired skin integrity
NURSING PRACTICE IV: Care of Clients with Physiologic and Psychosocial Alterations, Part B
November 17 & 18, 2018
33. The patient undergoes allergen testing using the cutaneous scratch method. To prevent anaphylaxis,
nurse Tina should initially monitor the patient’s ____.
A. Bilateral lung sounds
B. Arm at the site of the skin testing
C. Pupil’s size and reaction to ligh
D. Blood pressure and pulse

34. Nurse Tina knows that allergic reactions usually occur within minutes after injection of an allergen. The
nurse should ____.
A. Administer high flow oxygen
B. Document the patient’s allergy history
C. Observe the patient for about 20 minutes after injection
D. Monitor the patient’s blood pressure and pulse

35. The physician orders Epinephrine 1:10,000 0.5ml IV. After administering the drug, the NEXT action of
nurse tina is to ___.
A. Start oxygen at 100% using a non-breather mask.
B. Prepare an infusion of dopamine (Intropin)
C. Administer diphehydramine (Benadryl) IV.
D. Give a dose of cimetidine (Tagamet).

Situation - A 51 year old male is admitted for complaints of rectal bleeding, abdominal pain, weight loss
and change in bowel habits. Nurse Lawrence is aware that these are manifestations of colon cancer. The
physician prescribes radiation therapy and chemotherapy.

36. Nurse Lawrence knows that radiation therapy is used to treat colon cancer before surgery. Which of the
following is the effect of radiation therapy? It ____.
A. Help heal the bowl after surgery
B. Eliminates malignant cells
C. Cures the cancer
D. Reduces the size of the tumor

37. The patient undergoes radiation therapy. Nurse Lawrence noted that the patient’s white blood cell
(WBC) count is severely depressed. The PRIORITY nursing intervention would be to ____:
A. Place the patient in a private room and maintain strict aseptic technique for all procedures.
B. Instruct the patient to avoid shaving with a sharp razor
C. Encourage visitors to visit the patient regularly to reduce the feelings of isolation the patient may feel.
D. Encourage the patient to include fresh fruits and green leafy vegetables in his diet.

38. Nurse Lawrence is aware that chemotherapy should only be administered by nurses who have taken
special courses in administering chemotherapy and who are highly skilled. Before the nurse gives the
prescribed dose of chemotherapeutic agent, the nurse should do which of the following:
A. Verify the dose, drug and schedule with another trained nurse.
B. Collect an extra syringe and needle in case of contamination.
C. Explain the expected side effects of the drug to the patient
D. Cover the patient with a water resistant shield
NURSING PRACTICE IV: Care of Clients with Physiologic and Psychosocial Alterations, Part B
November 17 & 18, 2018

39. Nurse Lawrence assesses the patient receiving chemotherapy. Select the signs and symptoms that
would require further evaluation.
A. Patient complains of fatigue
B. Hair loss on scalp
C. Large areas of ecchymosis in various sites on the body.
D. Dry mucous membrane

40. Nurse Lawrence writes a care plan for the patient receiving chemotherapy. Select what should be
included in the care plan.

1. Nursing Diagnosis
2. Medical Diagnosis
3. Outcome
4. Interventions
5. Patient Education
6. Evaluation
A. 1, 2, 3, 4, 5
B. 1, 2, 3, 4, 6
C. 1, 3, 4, 5, 6
D. 2, 3, 4, 5, 6

Situation - Nurse Elma assists in the care of a 25 year old male who is admitted in the emergency
department for burns in the chest, abdomen, right arm and right leg.

41. The physician orders total parental nutrition (TPN) for the burn patient. Which of the following
statements is TRUE in this case? TPN is needed to _______.
A. Provide supplemental vitamins and minerals
B. Correct water and electrolyte imbalances
C. Ensure adequate caloric and protein intake
D. Allow the gastrointestinal tract to rest

42. Nurse Elma is aware that fluid shifts occur during the emergent phase of a burn injury. This shifting is
due to fluid moving from what space? From ________.
A. Intracellular to extracellular space
B. Extracellular to intracellular space
C. Vascular to intracellular space
D. Interstitial to vascular space

43. Nurse Elma understands that fluid shift results from an increase of which of the following?
A. Total volume of intravascular plasma
B. Total volume of circulating whole blood
C. Permeability of the kidney tubules
D. Permeability of capillary walls
NURSING PRACTICE IV: Care of Clients with Physiologic and Psychosocial Alterations, Part B
November 17 & 18, 2018
44. Which of the following fluid and electrolyte imbalance would Nurse Elma anticipate that the patient
would be particularly susceptible to in the emergent phase of burn care?
A. Hyperkalemia
B. Metabolic Alkalosis
C. Hemodilution
D. Hypernatremia

45. The patient is ordered to receive fluid resuscitation therapy. Nurse Elma adjusts the infusion rate by
evaluating the patient’s:
A. Hourly body temperature
B. Hourly urine output
C. Hourly urine specific gravity
D. Daily body weight

Situation - Nurse Flora admits a 30 year old female with tentative diagnosis of hypercalcemia.

46. Nurse Flora recognizes the signs and symptoms of hypercalcemia. Which of the following signs is an
indication of the diagnosis hypercalcemia?
A. Positive trousseau’s sign
B. Hyperactive bowel sounds
C. Hypertonicity of the muscles g form answer
D. Twitching

47. Nurse Flora recognizes the signs and symptoms of hypercalcemia. Which of the following signs is an
indication of the diagnosis hypercalcemia?
A. Muscle strength
B. Blood Pressure
C. Weight
D. Edema

48. The patient informs the nurse that she is taking Thiazide diuretics. Nurse Flora knows that Thiazide
diuretics drug are one of the most common causes of hypercalcemia. Which of the following signs should
the nurse observe for?
A. Increased peristalsis
B. Neurologic Depression
C. Neuromuscular Irritability
D. Decreased urine output

49. The patient goes into hypercalcemic crisis. Family members are anxious and worried. One relative
expresses to Nurse Flora “We don’t know what to do if she dies”. What is the BEST response of the nurse?
A. “Do not worry. We always see this kind of crisis and we can treat it.”
B. “I understand your concern, but I have to talk care of the patient first”.
C. “Yes, it is serious but I can come back, talk to you and answer your questions.”
D. “Has your loved one been eating processed food and drinking alcoholoic beverages lately?”
NURSING PRACTICE IV: Care of Clients with Physiologic and Psychosocial Alterations, Part B
November 17 & 18, 2018
50. Fortunately, the patient recovers from the hypercalcemic crisis. The patient is ready for discharge.
Which of the following activities should be included in the discharge plan? Instruct the patient to ____.
A. Take in anti-diarrheal medications as prescribed by the physician
B. Encourage foods that increase urine acidity
C. Decrease sodium and calcium intake
D. Restrict fluid intake to less than 1 liter a day

Situation - Mr. M.E., 37 years old, was accompanied by his wife and teen-age daughter to the out-patient
department for complaints of fever, fatigue, malaise and painful swollen joints. The physician ordered that
Mr. M.E. be admitted to the hospital for observation and treatment. You are the admitting nurse in the
OPD. You found out during the interview that Mr. M.E. does not have a regular job. His wife works as a
laundry woman.

51. Before admitting the client, you should FIRST make sure that:
A. The consent for admission is signed by the client.
B. The consent for admission is signed by the wife and witnessed by the daughter.
C. The client can pay his hospital bills.
D. The consent form is signed by the social worker.

52. Mr. M.E id brought to the medical ward. The next day, he wants to know about his illness. The nurse on
duty replied, “You don’t need to know your diagnosis”. Which of following rights of the patient is violated?
Right to ______.
A. Obtain from his physician complete current information concerning his diagnosis, treatment and
prognosis.
B. Receive from his physician information necessary to give informed consent.
C. Expect reasonable continuity of care.
D. Considerate and respectful care, irrespective of one’s socio - economic status.

53. After five days of hospitalization, the physician said Mr. M.E can be discharged. He ordered medications
to be taken at home. The client is still weak and symptomatic. Which of the following rights could be
violated in this case? Right to ____.
A. Know hospital rules and regulation
B. Privacy
C. Refuse treatment
D. Continuity of care

54. Because Mr. M.E cannot pay for his medical bills, he is referred to the social worker. Which of the
following rights is applicable on this case? The right to ___.
A. Considerate and respectful care irrespective of his socio-economics status.
B. Expect reasonable continuity of care
C. Examine and receive an explanation of his medical bills regardless of the source of payment
D. Know what hospital rules and regulation apply to his conduct as a client.

55. The nurse discusses and shares the medical records of Mr. M.E to a group of visiting members of a
medical mission team. Which of the following rights could be violated? The right to ___:
A. Expect that all communications and records pertaining to his care should be treated as confidential.
NURSING PRACTICE IV: Care of Clients with Physiologic and Psychosocial Alterations, Part B
November 17 & 18, 2018
B. Obtain information regarding any relationship of the hospital to another health care and educational
institution in so far as his care is concerned.
C. Informed consent
D. Privacy

Situation - Ms. Jenny is a charge nurse of an oncology unit. She prepares a unit plan to improve the delivery
of patient care and maximizing all human and material resources of the unit.

56. Ms. Jenny implements a change in the nursing delivery system from functional to team nursing. This
type of nursing model of practice is:
A. Nursing personnel are led by a registered nurse in providing care to a group of patients.
B. A task approach method used to provide care to patients.
C. Managed care concepts and tools are used in providing patient care.
D. A single registered nurse is responsible for providing care to a group of patients.

57. In implementing the change, Ms. Jenny has the consensus of the staff and other personnel involved in
the care of patients. The main purpose of achieving a consensus when making a decision within a group is
to ___.
A. Demonstrate that staff member are flexible
B. Facilitate cooperative effort toward goal achievement
C. Explore possible alternate solutions
D. Ensure the use of effective autocratic decision making

58. Ms. Jenny is aware that there are nursing and other personnel who may be resistant to the change and
are not taking an active part in facilitating the process of change. To overcome resistance to change, the
MOST important action by Ms. Jenny would be to ___.
A. Emphasize the positive consequences of the change
B. Identify the reason or reasons for the resistance
C. State clearly and concisely the purpose of the change
D. Modify the objectives to appeal to more people holding key positions

59. To ensure efficiency when organizing daily workload, Ms. Jenny reminds her staff that they should do
which of the following:
A. Organize care around legally required activities.
B. Plan activities to promote nursing convenience
C. Perform routine bed baths between 8:00 to 10:00 in the morning.
D. Provide care to a patient in isolation first.

60. Which of the following is the MOST effective resource in helping a staff nurse identify a solution to a
clinical problem?
A. Other staff nurses in the unit
B. Nursing procedure manual
C. Organizational chart of the hospital
D. Nurse manager of the unit
NURSING PRACTICE IV: Care of Clients with Physiologic and Psychosocial Alterations, Part B
November 17 & 18, 2018
Situation - A 55 year old male is admitted for complaints of joint pains, weakness, muscle spasm and
stiffness when waking up in the morning. He was diagnosed with Rheumatoid Arthritis (RA)

61. Nurse Ray is aware that RA is a debilitating chronic auto-immune disease. The patient asks the nurse
why his joints are becoming increasingly painful. Which of the following statements is TRUE about RA?
Rheumatoid Arthritis ____.
A. Is usually caused by the aging process
B. Results from degenerative joint damages
C. Begins with inflammation of joints synovial tissue
D. Affects only the weight bearing joints of the body

62. The patient further complains that he gets tired easily when helping his wife in household chores. He
further complains that whenever he walks, his knee hurts. Based on this information, Nurse Ray writes a
nursing diagnosis. Which of the following nursing diagnoses is MOST appropriate?
A. Activity intolerance related to fatigue and pain
B. Body image disturbance related to fatigue and pain
C. Ineffective individual coping related to chronic pain
D. Self- care deficit related to increasing joint pains

63. Nurse Ray develops a care plan for positioning the patient. Nurse Ray instructs the patient to assume a
position that would ___:
A. Prevent venous stasis
B. Promote maximum support
C. Prevent flexion deformities of the joints
D. Decrease edema around the joints

64. Nurse Ray writes a nursing diagnosis of activity intolerance related to lack of energy conservation.
Which of the following activities would Nurse Ray likely to choose to implement in response to the
diagnosis?
A. Encourage the patient to perform all tasks planned for the day.
B. Instruct the patient not to perform daily hygienic care until activity intolerance improves.
C. Administer narcotics to promote pain relief and rest
D. Encourage the patient to alternate periods of rest and activity throughout the day.

65. The patient is ready for discharge. He tell Nurse Ray, “I know it is important to exercise my joints, so I
won’t lose mobility. But my joints are so stiff and painful that exercise is difficult. “Which of the following
responses by Nurse Ray is MOST appropriate?
A. “Talk to your physician and tell him how you feel. Maybe he can increase the dosage of your pain
medication”.
B. “Take a warm bath before exercising. This may relieve some of your discomfort.”
C. “You are probably exercising too much. Decrease your exercise to every other day”.
D. “Stiffness and pain are part of your ailment. You can learn to cope by focusing on activities you enjoy”.

Situation - The nurse cares for several clients with acid- based imbalances.
NURSING PRACTICE IV: Care of Clients with Physiologic and Psychosocial Alterations, Part B
November 17 & 18, 2018
66. The nurse is aware that a client who has experienced cardiac arrest is MOST at risk for which of the
following imbalances?
A. Metabolic Alkalosis
B. Respiratory Alkalosis
C. Metabolic Acidosis
D. Respiratory Acidosis

67. The nurse assists in the care of a 30 year old male post surgical client undergoing nasogastric
suctioning. The nurse understands that clients with nasogastrics suction is MOST at risk with what
imbalances?
A. Respiratory Acidosis
B. Metabolic Alkalosis
C. Metabolic Acidosis
D. Respiratory Acidosis

68. The nurse assesses a 32 year old female client who appears very anxious, restless and irritable. The
client has marked increase rate and depth of respiration. Based on the information gathered, the client is
experiencing which of the following imbalances?
A. Respiratory Alkalosis
B. Metabolic Acidosis
C. Metabolic Alkalosis
D. Respiratory Acidosis

69. The female client who is very anxious and fidgety is blowing off to much carbon dioxide develops
tingling sensation of the lips and fingers and is not able to control her respirations. The most appropriate
nursing intervention for the client is to ___:
A. Instrust the client to blow her nose and take deep breaths
B. Administer oxygen
C. Have the client breathe into a paper bag
D. Administer intravenous fluids as ordered

70. The nurse obtains a sample of a client’s arterial blood gas (ABGs). Which of the following statements is
NOT true about ABGs?
A. Interpretation of the client’s ABGs involves evaluation of pH, PCO2 and HCO3; components of the ABGs.
B. ABGs assess the client’s oxygenation status and acid- base status.
C. ABGs provide information on blood parameters.
D. ABGs assess the client’s electrolyte and fluid balance.

Situation - Mila, a 35 year old female was brought in the Emergency Unit complaining of cold, clammy skin,
disorientation, restlessness and confusion. The nurse suspects the patient is developing cardiogenic shock.

71. Nurse Victor who admitted patient mila is aware that a decrease in cardiac output results in a decrease
in cerebral blood flow. Which of the following symptoms is one of the earliest sign of cardiogenic shock?
A. Tachycardia
B. Altered level of consciousness
C. Presence of fourth heart sound (S4)
NURSING PRACTICE IV: Care of Clients with Physiologic and Psychosocial Alterations, Part B
November 17 & 18, 2018
D. Decreased urine output

72. Nurse Victor assesses patient Mila. Decrease of which of the following factors would help detect that
the patient is at risk in developing cardiogenic shock?
A. Heart Rate
B. Cerebral blood flow
C. Cardiac Index
D. Blood Pressure

73. Nurse Victor reviews the medical history of patient Mila. Which of the following conditions is the MOST
common cause of cardiogenic shock?
A. Decreased hemoglobin level
B. Hypotension
C. Acute myocardial infarction (MI)
D. Coronary Artery Disease

74. Which of the following procedure would be most helpful to nurse victor to confirm the underlying
cause of cardiogenic shock?
A. Monitoring pulmonary artery pressure
B. Monitoring cardiac enzymes
C. Monitoring mean arterial pressure using an intra-arterial line.
D. Monitoring central venous pressure

75. The physician prescribes a treatment plan for patient Mila. Nurse victor understands that the initial
treatment goal is which of the following?
A. Prevent infection
B. Correct metabolic acidosis
C. Correct hypoxia
D. Increase myocardial oxygen supply

Situation – Ms. Dong-a, 35 years old, and a mother of two sought consultation at the OPD. She claims that
she felt a lump on her breast while doing a self-breast examination. On further examination, Nurse Atilde
confirmed that the lump has felt in the upper outer quadrant of the right breast.

76. The nurse evaluates nurse Dong-a for breast cancer. Which of the following is a risk factor for
developing breast cancer?
A. Using foam contraceptives
B. Having an early menarche before the age of 12
C. Having an early menopause before the age of 45
D. Giving birth to a first child before the age of 20

77. The nurse recalls that there are other known risk factors for breast cancer such as family history,
environmental and dietary factors. Which of the following is the BEST established dietary risk factor for
breast cancer?
A. Fat-rich foods
B. Alcohol intake
NURSING PRACTICE IV: Care of Clients with Physiologic and Psychosocial Alterations, Part B
November 17 & 18, 2018
C. Carbonated drink consumption
D. Caffeine consumption

78. The nurse is aware that for breast cancer patients, the single and MOST imporatant predictor of
outcome is the?
A. Presence of mastitis and an infection process upon diagnosis
B. Presence of nipple discharge at the time of diagnosis
C. Histological status of the axillary nodes
D. Age of the patient

79. Ms. Dong-a appeared to be worried and anxious. She ask the nurse what the doctor meant when told
that has Stage II cancer. The nurse explains that staging is based on the size of the primary tumor. In stage
II, the tumor is?
A. 3 cm in diameter and confined to the breast
B. Larger than 5 cm with involvement of the internal mammary lymph nodes
C. 2 cm or less in diameter and confined to the breast
D. Up to 5 cm or early metastasis to axillary lymph nodes

80. Based on further assessment, biopsy, results. Ms. Dong-a may have to undergo mastectomy. The nurse
recognizes that mastectomy is the treatment of choice when which of the following conditions apply?

1. Tumor involves the nipple-areola complex


2. Tumor is larger than 7 cm
3. Tumor exhibits extensive intraductal disease involving multiple quadrants of the breast
4. Patient cannot comply with daily radiation therapy
A. 1, 2, 3
B. 2, 3, 4
C. 1, 3, 4
D. 1, 2, 3, 4

Situation – Nurse Carol participates in a study the purpose of which is to determine the extent to which
gender, age weight and height predict selected physiologic outcomes, namely: forced expiratory volume in
one second, hemoglobin concentration, food intake and cancer-related weight change.

81. The independent variables of this study are the following, EXCEPT:
A. Height and weight
B. Age
C. Gender
D. Educational attainment

82. The dependent variables of this study are which of the following:
A. Cancer
B. Physical characteristics
C. Amount of serum concentration
D. Physiologic outcomes
NURSING PRACTICE IV: Care of Clients with Physiologic and Psychosocial Alterations, Part B
November 17 & 18, 2018
83. Nurse Carol formulates the problem statement for the study. She understand that a good problem has
which of the following characteristics: The problem?

1. Clearly and unambiguously identifies the variables under consideration


2. Clearly expresses the relationship of the variables to one another
3. Specifies the nature of the population being studied
4. Implies the possibility of empirical testing
A. 1, 3 and 4
B. 1 and 3
C. All of the options
D. 2 and 4

84. Nurse Carol determines the participants in the study. The sample of the study will be every 5th person in
the oncology unit. A table of random numbers will be used to select the beginning of the sample with in
the first sampling interval. This is an example of which sampling method?
A. Simple random sampling
B. Quota sampling
C. Stratified random sampling
D. Systematic sampling

85. Nurse Carol plans to use a method of data collection that use technical instruments to collect data
about the client’s physical, chemical, microbiological and anatomical status. Which of the following is a
method appropriate for the study?
A. Psychological
B. Physiological
C. Instrumentation
D. Pharmacological

Situation – You and your friends are enjoying a quiet meal in a restaurant. Suddenly you noticed that an
adult male customer was holding his throat and apparently choking on his food.

86. You approach the person who is apparently choking. What should you do FIRST?
A. Sweep the person’s mouth with your finger
B. Apply sharp upward thrusts over the person’s sternum
C. Determine if the person can make any verbal sounds
D. Hit the middle of the person’s back firmly

87. You prepare to perform the Heimlich maneuver on the conscious adult male. What are the steps to be
followed? Arrange the procedure in sequence?

1. Place the thump side of the first just above the umbilicus and well below and xiphoid process
2. Make a fist
3. Stand behind the victim
4. Place arms around the victim’s waist
5. Perform five quick in and up thrusts (between the umbilicus and xiphoid process)
A. 2, 5, 3, 4, 1
NURSING PRACTICE IV: Care of Clients with Physiologic and Psychosocial Alterations, Part B
November 17 & 18, 2018
B. 1, 2, 3, 4, 5
C. 4, 3, 1, 2, 5
D. 3, 4, 2, 1, 5

88. If the victim is unconscious, the sequence is:

1. Place the heel of one hand on top of the other between the umbilicus and xiphoid process
2. Straddle the victim’s thighs
3. Give five thrusts in and up with the heel of the bottom hand
4. Make a fist
A. 1, 2, 3
B. 1, 2, 3, 4
C. 2, 1, 3
D. 2, 4, 1, 3

89. When you perform abdominal thrusts during the Heimlich maneuver, you are trying to
A. Produce a burp to expel the foreign body
B. Pump the heart to push air out of the lungs
C. Produce a rush of air that expels the foreign body
D. Put pressure on the stomach to expel the foreign body

90. If the victim is pregnant or obese, which modification of the Heimlich maneuver would be appropriate
for this person?
A. Thrusts against the middle of the sternum rather than between the umbilicus and xiphoid process
B. Perform thrust gently rather than forcefully and discontinue the thrusts after 6 tries, if unsuccessful
C. Place the fist with the pinkle finger, rather than the thumb against the person’s body
D. Perform the thrusts with the person in the supine, rather than standing position

Situation – The nurse cares for a 40 year old female admitted with a diagnosis of chronic ulcerative colitis.
The nurse tries to communicate therapeutically with the client.

91. The nurse recalls that this client is her third admission in the last 8 months. The client is familiar with
the nurse. Which of the following remarks by the nurse would be MOST beneficial to the client?
A. “You are back again. What is wrong this time?”
B. “I don’t have to explain thigs to you. You have been here before anyway.”
C. “It’s been 3 months since you were last here. How do you feel about being back in the hospital?”
D. “It’s nice to see you again. Do you miss the hospital?”

92. The physician assesses the client and recommends a partial bowel resection and an ileostomy. The
client tells the nurse, “My doctor likes to play smart. I’m sure the more he operates, the better he likes it.”
Which of the following responses by the nurse would be MOST appropriate:
A. “Are you not being hard on your doctor? He is trying to help you.”
B. “What do you need by that?”
C. “You sound upset. We can talk about it, if you would like to.”
D. “Is your remark in any way have something to do with your operation?”
NURSING PRACTICE IV: Care of Clients with Physiologic and Psychosocial Alterations, Part B
November 17 & 18, 2018
93. The client become increasingly irritable thinking about her operation. She is unpleasant to her visitors
and refuses her medication and treatments. Which of the following nursing interventions would be
therapeutic?
A. Encourage the client to direct her anger to her family members
B. Continue performing assigned tasks and duties as though nothing has happened
C. Encourage the client to discuss her feelings and concerns
D. Offer the client positive reinforcement everytime she cooperates with her medications and treatments

94. The client refuses to eat her lunch and angrily tells the nurse to get out of her room. What would be the
nurse’s best response?
A. “I’ll get you something to relax you.”
B. “You sound angry. What is upsetting you?”
C. “Your anger does not bother me. I’ll back later.”
D. “I’ll leave, but you need to eat.”

95. The nurse makes arrangement for a former client who underwent ileostomy to talk with the client. The
reason for which is to:
A. Encourage the client and provide realistic information on ileostomy
B. Let the client know that she is not the only one who experienced the operation
C. Provide support for the plan of the physician regarding therapy of the client
D. Convince the client that she will not be disfigured and can still lead quality life

Situation – Ms. Ada is a nurse working in the surgical unit. She is aware her legal responsibilities as she
assists in the care of post-operative patients.

96. Which principle is applicable in a situation where a sponge was left inside the abdomen of a patient
who had an exploratory laparotomy?
A. Doctrine of Force Majeure
B. Doctrine of Res Ipsa Loquitor
C. Doctrine of Viz major
D. Doctrine of Respondeat Superior

97. Under the Doctrine of Respondeat Superior, who among the following is liable if the patient who had
exploratory previously was reopened and a piece of gauze was found in the abdominal cavity? The:
A. Instrument nurse
B. Surgeon
C. Operating room nurse
D. Assistant surgeon

98. The Doctrine of Respondeat Superior holds that:


A. The employer is responsible for the actions of his/her employee
B. The employer is not responsible for the action of his/her employee
C. The employee is not responsible for his/her own action
D. Using restraints without the patient’s permission constitue
NURSING PRACTICE IV: Care of Clients with Physiologic and Psychosocial Alterations, Part B
November 17 & 18, 2018
99. Performing an act which a reasonable and prudent nurse would not do or the failure to perform an act
which a reasonable and prudent nurse would have done under similar situation is construed as:
A. Misdemeanor
B. Malfeasance
C. Malpractice
D. Negligence

100. Ms. Ada learns of a nurse in the orthopedic ward who applied hot water bottle over a paralyzed leg
which consequently burned. This is an example:
A. Malfeasance
B. Misdemeanor
C. Malpractice
D. Negligence
NURSING PRACTICE IV: Care of Clients with Physiologic and Psychosocial Alterations, Part B
November 17 & 18, 2018

KEY ANSWER FOR NURSING PRACTICE IV

1.C 11.A 21.D 31.B 41.D 51.A 61.C 71.B 81.D 91.C
2.B 12.B 22.A 32.D 42.C 52.A 62.A 72.C 82.D 92.C
3.D 13.D 23.C 33.A 43.D 53.D 63.N/A 73.C 83.C 93.C
4.C 14.A 24.B 34.C 44.A 54.A 64.C 74.D 84.D 94.B
5.B 15.D 25.C 35.A 45.B 55.D 65.B 75.D 85.B 95.A
6.B 16.A 26.C 36.D 46.N/A 56.A 66.C 76.B 86.C 96.B
7.C 17.D 27.D 37.A 47.A 57.B 67.B 77.A 87.D 97.B
8.B 18.A 28.A 38.A 48.B 58.B 68.A 78.C 88.D 98.A
9.D 19.A 29.D 39.C 49.B 59.B 69.C 79.A 89.C 99.D
10.D 20.B 30.B 40.B 50.B 60.B 70.D 80.D 90.A 100.C
NURSING PRACTICE V: Care of Clients with Physiologic and Psychosocial Alterations, Part C
November 17 & 18, 2018
Situation - You’re assigned as a staff nurse in the Stroke Unit of the Medical Ward. One of your
responsibilities is to provide health teachings to the stroke patients and to the other members of the
family. The following questions refer to these statements.

1. You are teaching a patient about taking prophylactic warfarin sodium (Coumadin). Which of these
statement indicate that the patient understands how to take the drug. SELECT all that apply:

1. The drug’s action peak in two hours


2. Maximum dosage is not achieved until 3 to 4 days after starting the medication
3. Effects of the drug continue 4 to 5 days after discontinuing the medication
4. Protamine Sulfate is the antidote for Warfarin
5. I should have my blood levels tested periodically
A. 2, 4, 5
B. 2, 3, 5
C. 1, 4, 5
D. 1, 3, 4

2. Maintaining oral hygiene for a stroke patient is important in his care. You are teaching a family member
how to give oral care to the stroke patient. Which of the following nursing measure is NOT APPROPRIATE
when giving oral care to such patient?
A. Cleaning the patient’s mouth and teeth with a soft-bristled toothbrush
B. Keeping portable suctioning equipment at the bedside
C. Opening the patient’s mouth with a padded tongue depressor
D. Placing the patient on his back with a small pillow under the head

3. Your patient has paraplegia. In giving instruction to the watcher when changing the patient’s position in
bed, which of the following is NOT APPROPRIATE?
A. Lifting the patient when moving him up in bed
B. Rolling the patient unto the side
C. Using a trapeze to help the patient lift off the bed
D. Sliding the patient to move up in bed

4. A patient is experiencing mood swings after stroke. Often times she has episodes of crying that are
distressed to the family members. What is the BEST TECHNIQUE that you will advise the family members
when the patient is having crying spells?
A. Ignore the patient during this time
B. Sit quietly with the patient until the crying episode is over
C. Tell the patient that the behavior is unacceptable
D. Attempt to divert the patient’s attention

5. To prevent joint deformities of the arm and hands of a hemiplegic patients what are the APPROPRIATE
position will you include in your health teachings of the family member? SELECT all that apply.
A. 2, 3, 4
B. 1, 3, 5
C. 1, 2, 4
D. 2, 4, 5
NURSING PRACTICE V: Care of Clients with Physiologic and Psychosocial Alterations, Part C
November 17 & 18, 2018

Situation – You are a community health nurse assisting a family in the care of their daughter suffering from
the post traumatic stress disorder

6. While caring for this client, the family notices that loud noises cause a serious anxiety response. Which
of the following explanations by you would help the family understand the client’s response?
A. After a trauma, the client cannot respond to stimuli in an appropriate manner
B. Clients often experience extreme fear about normal environmental stimuli
C. Environmental triggers can cause the client to react emotionally
D. The response indicates that another emotion problem needs investigation

7. Which of the following instructions should you include about relationships for this client with
post-traumatic stress disorder?
A. Assess the client’s discomfort when talking about feelings to family member.
B. Explain that avoiding emotional attachment protects against anxiety.
C. Warn the client that she will have a tendency to be over dependent in relationships
D. Encourage the client to resume former roles as soon as possible

8. Which of the following nursing intervention would best help this client and her family handle
interpersonal conflict at home? Have the family:
A. Discuss how to change dysfunctional family patterns
B. Teach the client to identify defensive behavior.
C. Agree not to tell the client what to do about problem
D. Arrange for the client to participate in social activities.

9. Which of the following nursing actions would be more appropriate when speaking with this client about
the trauma she experienced?
A. Listen attentively
B. Request the client write what is being said
C. Ask question to convey an interest in the details
D. Obtain validation of what the client says from another party

10. Which of the following nursing actions would you include in the plan of care for this client who keeps
saying that the trauma she experienced was just a case of bad luck?
A. Assist the client in defining the experience as a trauma
B. Encourage the client to verbalize the experience
C. Work with the client to accept positive and negative feelings
D. Help the client accept positive and negative feelings

Situation – One of the competencies is expected of you as a beginning professional nurse is to have a
positive attitude toward research

11. You plan to study the effectiveness of films in teaching active exercises to post stroke patients. Which of
the following research designs will give you the most confidence to answer the research problem?
A. time- series design
B. non-equivalent control group design
NURSING PRACTICE V: Care of Clients with Physiologic and Psychosocial Alterations, Part C
November 17 & 18, 2018
C. one-group pre-test, post-test design
D. post-test only, control group design

12. Which of the following sampling techniques is the most appropriate to answer a research question on
the effectiveness of active exercise on the mobility of post stroke patients?
A. convenience sampling
B. purposive sampling
C. quota sampling
D. random sampling

13. You want to develop a tool for measuring the level of consciousness of stroke patients. This is what type
of research?
A. Historical study
B. Case study
C. Comparative study
D. Methodological study

14. You want to examine the difference in the eating behaviors between two groups of high school
students. Which of the following research designs would you use to answer the research question?
A. Case study
B. Correlation study
C. Comparative design
D. Survey study

15. In a study on the relationship between personality make-up and obesity, a Pearson r=.8 would mean:
A. a high, negative correlation
B. a moderate, positive correlation
C. a high, positive correlation
D. a low, positive correlation

Situation – Ella, a 10-year old grade four pupil in a private school has just lost her best friend, Lory, who
died of Leukemia. Her mother brought her to the Emergency Room who reported that Ella has been
gloomy and often expresses feeling of joining Lory in heaven.

16. Which of the following is the precipitating factor for Ella’s feeling of wanting to die?
A. The death of Lory
B. Her age
C. Her authoritative mother
D. Her fear of death

17. Ella’s feeling of joining Lory in heaven is manifestation of:


A. Suicidal intent
B. Suicidal threat
C. Paranoia
D. Suicidal ideation
NURSING PRACTICE V: Care of Clients with Physiologic and Psychosocial Alterations, Part C
November 17 & 18, 2018
18. Ella state “I will hang myself”. This is a manifestation of:
A. Denial
B. Suicidal threat
C. Suicidal intent
D. Paranoia

19. Which of the following is the best predictor of adolescents attempting suicide?
A. Depressed mood
B. Joyful mood
C. Feeling of euphoria
D. feeling of hopelessness

20. Ella state “I wish I were dead. I cannot stand anymore not having Lory around.” Your most appropriate
nursing action would be:
A. Do nothing because Ella will not do it
B. Refer Ella to your supervisor
C. Stay with Ella
D. Explore Ella’s feeling

Situation – You are a staff nurse in the Out-Patient Department. Mrs. D brought her 9-year old daughter
Bianca for consultation, Bianca is complaining of sore throat, muscle tenderness, weakness of the arms and
a general feeling of not being well. The attending physician’s impression is infectious Polyneuritis.

21. Initially, what assessment data would be the BEST IMPORTANT for you to make?
A. Exposure to an infectious disease
B. Difficulty in swallowing
C. Difficulty in urinating
D. Diet intake for the last 24 hours

22. Bianca was admitted to the Pediatric ward. What would be your PRIORITY nursing action when caring
for her?
A. Providing a diversional activity for Bianca
B. Evaluating Bianca’s bilateral muscle strength
C. Making a game of range-of motion exercise
D. Assessing Bianca’s ability to follow simple commands

23. The pediatric ward nurse ask Bianca to cough every now and then. She also assesses Bianca’s speech for
decreased volume and clarity, the underlying rationale for these assessments is to determine the
________.
A. regression to an earlier developmental phase
B. involvement of facial and cranial nerves
C. increased intracranial pressure
D. inflammation of the larynx and epiglottis
NURSING PRACTICE V: Care of Clients with Physiologic and Psychosocial Alterations, Part C
November 17 & 18, 2018
24. Assessment findings revealed that Bianca has weak gag and cough reflexes. During this acute phase of
Bianca’s illness, which of the following problems should receive the HIGHEST PRIORITY?
A. Impaired swallowing related to neuromuscular impairment.
B. Risk for infection due to an altered immune system
C. Ineffective breathing pattern to neuromuscular impairment
D. Total urinary incontinence related to fluid losses

25. Bianca was placed on a medical ventilator. Which of the following nursing actions should the ward
nurse give emphasis to?
A. Turning Bianca slowly and gently from side to side to prevent respiratory complication
B. Engaging Bianca in vigorous passive range-of-motion
C. Maintaining Bianca in a supine position to prevent unnecessary nerve stimulation
D. Transferring Bianca to a bedside chair three times a day to prevent posture hypotension

Situation – Jennylyn Abad, 40 years old woman, has a history of Rheumatoid Arthritis for the last ten years.
The following refer to this situation.

26. Which of the following is common sign/symptom of rheumatoid arthritis?


A. Presence of crepitus over joints
B. Symptoms aggravated by humidity
C. Deformity and displacement of proximal joints
D. Presence of Heberden’s nodes

27. Which of the following would NOT be a common laboratory findings in rheumatoid arthritis?
A. Low hemoglobin
B. Positive RA factor
C. Positive Lupus Erythematosus (LE) prep
D. Increase white blood count

28. Jennylyn has been taking steroids for the last five years to control her arthritis. Which of the following is
a common side effects of steroid therapy?
A. Hyponatremia
B. Hyperkalemia
C. Increase blood glucose
D. Protein anabolism

29. Which of the following diet is best suited to decrease the side effects of Jennylyn’s steroid therapy?
A. Normal carbohydrate, high protein, high potassium
B. Low carbohydrate, high protein, high potassium
C. Low carbohydrate, high protein, low potassium
D. High carbohydrate, low protein, low potassium

30. Jennylyn was on Ibuprofen which has been stopped when the steroid therapy was started. Which was
the main reason for stopping the Ibuprofen once the steroid therapy was started?
A. The two drugs taken together increase the risk of GI bleeding
B. The Ibuprofen would interfere with the action of the steroid
NURSING PRACTICE V: Care of Clients with Physiologic and Psychosocial Alterations, Part C
November 17 & 18, 2018
C. The Ibuprofen was not needed with the steroid
D. The two drugs together would cause severe liver damage

Situation – You are a staff nurse in the Musculo-skeletal Unit of the hospital. You are taking care of the
patients with varied Musculo-skeletal conditions.

31. Which of the following inflammatory conditions commonly occurs in the shoulder?

1. Bursitis
2. Carpal Tunnel Syndrome
3. Tendonitis
4. Dupuy trens
A. 1 and 2
B. 2 and 3
C. 3 and 4
D. 1 and 3

32. Which of the following is NOT considered a conservative treatment for inflammatory conditions of the
shoulder?
A. Non-steroid Anti-inflammatory Drugs
B. Intermittent ice and heat application to the joint
C. Laser photo-therapy
D. Resting of the extremity

33. Impaired movement of the rotator cuff of the shoulder is generally termed as Impingement Syndrome.
Edema, which is an early manifestation of this syndrome, is due to ________.
A. acute trauma of the shoulder
B. muscle spasm
C. hemorrhage of the structures involved
D. atrophy due to disuse

34. For a persistent shoulder pain and weakness, this usual treatment modality performed is _______.
A. Pulse-magnetic field
B. Laser photo-therapy
C. Radio-frequency ablation
D. Arthroscopic Synovectomy

35. One of the most common foot problems is Callus. This condition precedes the formation of a Callus
_____.
A. External pressure on the foot
B. Faulty foot mechanics
C. Flexion deformity of the interphalangeal joint
D. Penetrating nail plate in the surrounding skin of the foot

Situation – You are a staff nurse working in a hospital and encounter situations with ethico-moral
implications.
NURSING PRACTICE V: Care of Clients with Physiologic and Psychosocial Alterations, Part C
November 17 & 18, 2018

36. Which of the following nursing actions will you do if the patient is unconscious and unable to make
decisions? This fact should be:
A. reported to the nurse supervisor
B. reported to the attending physician
C. documented in the patient’s chart
D. reported to the police

37. A patient came in from prison for emergency treatment. Which of the following precautionary
measures will you institute to ensure the safety of the hospital staff and the other patients?
A. Provide the same assessment and care to the client
B. Use restraints
C. Lock the ER door against entry if security is in question
D. Never release the hand or ankle restraint

38. In using restraints to patients, which of the following guidelines will you follow?
A. Apply restraints because the patient needs it
B. Apply restraints according to the hospital policy
C. Restraint is the most effective nursing intervention to minimize aggressive behavior
D. Maintain respect for the client when applying restraints

39. You have observed that a group of students are implementing their research project on the effect of hot
and cold application to febrile patients without informing the participants properly. What ethical principle
was violated by the students?
A. Nonmaleficence
B. Autonomy
C. Justice
D. Respect for person

40. A researcher should pay particular attention in protecting the rights of certain vulnerable groups. Which
of the following is NOT considered vulnerable?
A. Prisoners
B. Children
C. Mentally-challenged persons
D. Postpartum mother with normal delivery

Situation – You are a staff nurse in the Neurologic Unit of the Pediatric Department. Several problems have
been reported in relation to ventriculo-peritoneal shunt for hydrocephalus patients. The initial investigation
on the care of patient with this shunt revealed that majority of the staff nurses have little experience on
caring for such patients. It was decided to look into their level of knowledge and skills to serve a baseline
data for improving the nursing care of hydrocephalic patients on ventricular-peritoneal shunt. The following
questions refer to this situation.

41. A four-year old hydrocephalic preschooler is scheduled to have a ventriculo-peritoneal shunt at the
right side of the head. Which of the following positions should the child be placed immediately after
surgery?
NURSING PRACTICE V: Care of Clients with Physiologic and Psychosocial Alterations, Part C
November 17 & 18, 2018
A. Supine, with the head of the bed flat
B. On the left side with the head of the flat
C. Prone, with the head of the bed elevated
D. On the right side, with the foot of the bed elevated

42. What nursing action should the nurse do when providing post-operative nursing care to a child after
insertion of a ventriculo-peritoneal shunt?
A. Monitoring for the increased temperature
B. Check the urine for glucose and protein
C. Administer narcotics for pain control
D. Test cerebrospinal fluid leakage for protein

43. An infant who has hydrocephalus is irritable, lethargic, and difficult to feed before the
ventriculo-peritoneal shunt is placed. To maintain the infant’s nutritional status, which of the following
nursing action is the MOST APPROPRIATE?
A. Feeding the infant just before doing any procedures
B. Giving the infant small, frequent feedings
C. Scheduling the feeding every six hours
D. Feeding the infant in a horizontal position

44. The parent of a school-age child with ventriculo-peritoneal shunt were given discharge teaching. Which
of the following signs of a blocked shunt would the parents be able to identify that the discharge teaching
is successful?
A. Irritability and increase difficulty with eating
B. Elevated temperature and reddened incisional site
C. Decreased urine output with stable intake
D. Tensed fontanel and increased head circumference

45. After a six-year old child underwent placement of ventriculo-peritoneal shunt, he was put on I.V.
medication. The head nurse observed that many of the staff nurses are disconnecting the flush syringe first
and then clamping the intermittent infusion device. The staff nurse seem not to understand the benefits of
positive pressure techniques. Which of the following strategies would be the MOST EFFECTIVE way to
improve the staff nurses technique in giving I.V. medication?
A. Post an evidence-based article on the topic in the unit
B. Create a poster presentation on the topic with a required post test
C. Send a memorandum on the importance of clamping the device first
D. Ask each nurse if they are aware that their practice is not current

Situation – You are a staff nurse in the Stroke Unit of the Medical Ward. You have a stroke patient with a
tracheostomy. In the Unit’s last meeting it was discussed that there has been increasing complaints on the
nurses incompetence in caring for a patient with tracheostomy. The following questions relate to this
situation.

46. In providing tracheostomy care which of the following is the nurse’s PRIORITY nursing action? The nurse
_____.
A. cut the dressing using sterile scissors
NURSING PRACTICE V: Care of Clients with Physiologic and Psychosocial Alterations, Part C
November 17 & 18, 2018
B. cleans the incision with iodine-based antiseptic
C. secure clean ties before removing soiled ones
D. used clean technique

47. Which of the following nursing actions should the nurse TEACH the patient’s care giver regarding
tracheostomy care?
A. Remove the tracheostomy tube if obstruction occurs
B. Leave the tracheostomy ties in place until the physician change them
C. Remove and clean the inner cannula daily
D. Never put a covering over the tracheostomy stoma

48. After a tracheostomy procedure, the nurse documents important observations made during the
procedures such as _______.
A. response of the patient after the procedure
B. vital signs of the patient during the procedure
C. amount, color, and consistency of sputum and appearance of the incision
D. behavior of the patient during the procedure

49. What is the purpose of a tracheostomy? To _____.


A. establish an airway
B. establish a pathway for nutrition
C. monitor respiratory functions
D. maintain the patency of the airway

50. Which of the following can be a MAJOR PROBLEM for a patient with tracheostomy?
A. Breathing
B. Ambulating
C. Swallowing
D. Singing

Situation - You are staff nurse in the Psychiatric Unit of a private hospital. Lorena, a 20-year old stage
actress was admitted with the chief complaints of getting angry easily and inability to tolerate being alone.
She claimed that she has also the tendency to manipulate people and feels unhappy most of the time. She
was diagnosed to be suffering from Borderline Personality Disorder (BPD).

51. You have observed that Lorena is manifesting “spitting”. This characteristic of BPD is BEST defined as
______.
A. viewing people and objects as parts, either good or bad
B. having two personalities
C. talking about other people behind their back
D. literally spitting in other people’s face

52. When assessing a patient with BPD which of the following information would you focus on?
A. Ability to get people on his/her side
B. Disruption in some aspect of his/her life
C. desire for intimate relationship
NURSING PRACTICE V: Care of Clients with Physiologic and Psychosocial Alterations, Part C
November 17 & 18, 2018
D. Increase acceptance from other people

53. Lorena revealed she has a tendency to manipulate others. What would be the MOST APPROPRIATE
short-term goal for her? For Lorena to _____.
A. have an intimate relationship
B. acknowledge her own behavior
C. stop arguing with other people
D. express her feelings verbally

54. Patient with BPD manifest transient psychotic symptoms. What is the drug of choice to treat these
symptoms?
A. Mood stabilizer
B. Benzodiazepines
C. Lithium
D. Antipsychotics

55. Lorena has a history of alcohol abuse so she was started on Antabuse. Which of the following is a
COMMON side effect of this drug?
A. Hypertension
B. Bradycardia
C. Depression
D. Elation

Situation - Angela, a 40-year old, single mom of two preschool children, aged two (2) and five (5) years old
respectively was admitted to the Psychiatric Unit for attempting to kill her two children.

56. Angela says “My dead mother ordered me to kill my two children before the devil could get them”.
What behavior is being manifested by Angela?
A. Auditory hallucination
B. Visual hallucination
C. Tactile stimulation
D. Olfactory hallucination

57. You ask Angela to touch a 25-centavo coin and a one-peso coin. You have observed that Angels was
having difficulty differentiating one coin from the other. This is manifestation of _____.
A. Astereognosis
B. Kinesthetic
C. Agraphestesia
D. Cenesthetic

58. You have observed that Angela is manifesting persistent hallucination. What is the MOST EFFECTIVE
therapy in treating patient with a condition like that of Angela’s?

1. Cognitive – behavior therapy


2. Crisis intervention
3. Psycho- pharmacologic treatment
NURSING PRACTICE V: Care of Clients with Physiologic and Psychosocial Alterations, Part C
November 17 & 18, 2018
4. Anti – psychotic drug
A. 3 and 4
B. 1 and 2
C. 2 and 4
D. 1, 2, and 3

59. Which of the following is the GOAL of nursing intervention for a patient who is hallucination.
A. To help the patient to have an increased awareness that in symptoms she is experiencing is not real
B. To help the patient to identify what triggers her hallucination
C. To encourage the patient to verbalized her fear, anxiety, and anger
D. To help the patient to go back to her real world

60. What communication technique would be MOST effective to achieve the nursing goal for patient with
hallucination?
A. Listening
B. Facilitative communication
C. One-on-one discussion
D. Social interaction

Situation - Charting is part of the professional nurse’s responsibility related to record management. The
following questions refer to this statement.

61. One of the characteristics of charting is brevity. Which of the following example of charting shows this
characteristics?
A. Nurse brought the patient to OR via stretcher at 10:15
B. Patient left for surgery via stretcher at 10:15 am
C. To surgery via stretcher at 10:15
D. patient brought by the nurse to OR via stretcher at 10:15 accompanied by the “bantay”

62. Which of the following is NOT a requirement for a late entry in charting?
A. nurse’s initials above the time
B. may be made if something has been forgotten
C. circle it and write “late entry”
D. reason for the late entry

63. There is a blank space after your last entry in your charting. You will ______.
A. draw a double line after your last entry
B. sign your full name after your last entry
C. draw a horizontal line through the center of an empty line
D. draw a perpendicular line across the empty space

64. Which of the following should the nurse’s notes focus on?
A. Immediate past and the present
B. The present only
C. The future
D. The recent past
NURSING PRACTICE V: Care of Clients with Physiologic and Psychosocial Alterations, Part C
November 17 & 18, 2018

65. Which of the following sample charting would show the characteristic of ACCURACY?
A. intake from 700-1000 ml: 80 ml of coffee; 240 ml of orange juice; 500ml of water
B. Patient on forced fluid but refused to take it most of the time
C. Patient on forced fluids observed
D. Given fluid at frequent intervals but takes only a few sips

Situation - During summer season incidence of heat stroke rises. As a community health nurse you should
disseminate information on this as part of your role as a health educator.

66. A patient came in with sign of heat stroke. Which of the following are signs/symptoms of profound CNS
dysfunction?
A. Elevated temperature
B. Hot, dry skin
C. Tachypnea, hypotension, tachycardia
D. Confusion, delirium, bizarre behavior

67. Which of the following is the primary goal in the care of patient with a heat stroke?
A. To maintain cardiac functions
B. To reduce immediately the high temperature
C. To restore normal body temperature
D. To prevent further complication

68. Which of the following is the best advice you would give to an athlete to prevent heat stroke?
A. To monitor fluid losses and weight lost during workout activities and to replace fluids.
B. To avoid immediate exposure to high temperature
C. To maintain adequate fluid intake
D. To avoid planning outdoor activities between 10 a.m. and 2 p.m.

69. To prevent heat stroke, the following pieces of advice are given to the community, EXCEPT:
A. Avoid direct exposure to the sun especially late in the morning
B. Always drink plenty of water
C. Use a hat or an umbrella when going out of the house
D. Just stay home and relax

70. Which of the following would your best advice to the community people during summer time to
prevent dehydration?
A. Use a hat or an umbrella when going out of the house
B. Just stay home and relax
C. drink plenty of fluids
D. wear light-colored clothing

Situation - Lizbeth, a 30-year old registered nurse with two children, legally separated from her husband
was admitted to the psychiatric unit three weeks ago.
NURSING PRACTICE V: Care of Clients with Physiologic and Psychosocial Alterations, Part C
November 17 & 18, 2018
71. You are the nurse attending to Lizbeth. You have observed that she has a habit of washing her hands
repeatedly for a long period of time. This is a manifestation of what kind of behavior?
A. Negative
B. Hyperactive
C. Ritualistic
D. Nonconformist

72. Lizbeth engages in this behavior to _____.


A. protect herself from undesirable people
B. relieve her anxiety
C. occupy herself with purposeful activity
D. call the attention of other

73. A new nurse introduce herself to Lizbeth and asks her name. Lizbeth responds “I am an obsessive-
compulsive neurotic. I have had psychoanalysis for ten years. What do you think can you do for me?” Your
BEST response would be _____.
A. “Can we talk about that Lizbeth?”
B. “I need to know you better Lizbeth”
C. “You seem to feel hopeless”
D. “who was your psychoanalyst?”

74. Lizbeth tells you “That the new nurse makes me angry. Like you, she does not understand what my real
problem is”. Your BEST reply would be _____.
A. “You seems to be upset. I will come back later”
B. “You have the right to be upset when people don’t seem to understand”
C. “That’s a common feeling. I understand. Let’s talk about it”
D. “I know what your problem is. You are an obsessive-compulsive personality”

75. Diazepam (Valium) was prescribe for Lizbeth. You gave her instructions on effects of the drug. What
statement would indicate that Lizbeth needs further health teaching about the medication?
A. “I’m so glad I can still eat chocolate while I’m taking this.”
B. “I’m so glad no blood tests are necessary while I’m taking this.”
C. “I’m so glad Valiums won’t affect my driving skills”
D. “I’m so glad I will only have to take this until I learn to be less anxious”

Situation - As a professional nurse, one of your roles is an advocate for the patient’s rights. The following
questions refer to this statement.

76. You were invited by a women’s organization to be the resource speaker on “Violence against women
and children”. Women and their children are protected against violence under Republic Act No.__.
A. 9173
B. 9211
C. 9262
D. 9160

77. One of the violence acts is sexual violence. Which of the following does NOT constitute sexual violence?
NURSING PRACTICE V: Care of Clients with Physiologic and Psychosocial Alterations, Part C
November 17 & 18, 2018
A. Acts of lasciviousness
B. Rape
C. Treating a child as a sex object
D. Economic abuse

78. What would be the nurse’s MAIN concern when a case of abuse, maltreatment, or neglect is suspected
in a patient?
A. Reporting the incident to proper authorities
B. Safety and welfare of the patient
C. Referring the patient to a mental health worker
D. Just keep quiet about the matter

79. What is the MOST common clinical manifestation of child neglect?


A. Missed appointment with health care provider
B. Fracture
C. Unexplained bruises
D. Malnutrition and dehydration

80. What should be the FOCUS of care for patients who are suspected to be abused, maltreated, or
neglected?
A. Immediate treatment of any injury
B. Consequences of the act
C. Prevention of further injury
D. Referral to legal authorities

Situation - You are a staff nurse at the Ear Unit of the Medical Ward of a government hospital. Clarita, a
25-year old stage actress was admitted with the chief complaint of on and off tinnitus at the right ear. The
audiogram confirms conductive hearing loss or mixed loss especially in the low frequencies. Clarita was
diagnosed with Otosclerosis.

81. Otosclerosis is a common cause of what condition?


A. Premature labor
B. Meningitis Auditory nerve
C. Sensori-neural hearing loss
D. Conductive hearing loss

82. The audiogram also confirms a sensori-neural hearing loss. What ear structure is damaged when this
condition is present?
A. Outer and/or middle ear
B. Tympanic membrane
C. Cochlear nerve
D. Stapes

83. Clarita was given Sodium Flouride. What is the rationale for giving this to her?
A. To mature the spongy bone growth
B. To remove the diseased stapes
NURSING PRACTICE V: Care of Clients with Physiologic and Psychosocial Alterations, Part C
November 17 & 18, 2018
C. To prevent further complications
D. To restore hearing

84. A surgical procedure was recommended for Clarita. Which of the following surgical procedures is useful
in correcting her condition?
A. Myringotomy
B. Ossicular reconstruction
C. Cochlear transplant
D. to restore hearing

85. After her surgery, Clarita was placed on her left side with the head of the bed elevated. What is the
rationale for placing her in this position?
A. To minimize the pressure in the middle ear
B. To prevent complication of bleeding
C. To ensure patient safety
D. To prevent vomiting

Situation - Ms. Ligaya Co is a chief nurse in a secondary level hospital with nursing staff consisting of
registered staff nurses, nursing assistants and aides.

86. Ms. Co stresses the importance of promoting ‘esprit d corps’ among the nursing staff. Which of the
following statement indicates that they understand the meaning of the term?
A. “In order that we achieve the goals of the institute, we must follow the directives coming from above.”
B. “We will ensure that all resources we need are available when needed”
C. “Let’s work together in harmony; we need to be supportive of one another.”
D. “We need to show our competence to the higher management level.”

87. She assert the importance of promoting a positive organizational culture among the nursing staff.
Which of the following behaviors indicate that this has been attained?
A. Obedient and uncomplaining
B. Powerful and oppositional
C. Competitive and perfectionist
D. Caring and nurturing with one another

88. Ms. Co is a visionary strategist, and desires to be a committed leader. Which of the following types of
leadership do these behaviors reflect?
A. Servant leadership style of politicians
B. Transactional structure, order of the org
C. Connective connecting people
D. Transformational

89. She knows that as a leader, she has to strategize in order to create followership in response to
authority. This capacity to act or the strength to accomplish a goal is referred to as _____.
A. power
B. accountability
C. responsibility
NURSING PRACTICE V: Care of Clients with Physiologic and Psychosocial Alterations, Part C
November 17 & 18, 2018
D. authority

90. Ms. Co gathered the nursing staff and other health care worker in the hospital who support the idea of
having a meeting to discuss problems that affect the delivery of care and to agree to speak with one voice.
This power mechanism is called ______.
A. coalition
B. upward appeal
C. rationality
D. assertiveness

Situation - Ana, a 28-year old supervisor in a business processing office, consult at the Out-patient
department of the hospital with the chief complaints of headache, drowsiness, nausea, and vomiting. She
revealed that one week before consult she stopped drinking coffee which she used to take three to five
times a day. She was diagnosed with Caffeine Discontinuation Syndrome (CDS).

91. What are the effects of drinking coffee two to three cups a day on the central nervous system?
A. Feeling motivated and energized, less fatigue
B. Feeling motivated and energized, headache
C. Feeling motivated and drowsy, headache
D. Headache, less fatigue

92. Which of the following reinforces a person to continue drinking coffee?


A. Feeling of unpleasant symptoms when drinking is abruptly stopped.
B. Drinking coffee at Starbucks is the “in-thing
C. Feeling of well-being
D. The price of coffee is getting lower

93. Ana was given Luminal 60 mg./day p.o. this drugs is a/an _____:
A. intermediate-acting sedative
B. short-acting sedative
C. hypnotic drug
D. long-acting sedative

94. While doing your assessment Ana complained of nausea. Which of the following would be you
PRIORITY nursing action?
A. Refer Ana to the attending physician immediately
B. Let Ana put her head in-between her legs
C. Administer an anti-emetic drug
D. Offering Ana some ice chips

95. You have observed Ana to be pacing in the room, restless, and stutters when your speaking to her. You
would assess Ana as manifesting which of the following global anxiety response?
A. Cognitive
B. Behavioral
C. Motor
D. Biological
NURSING PRACTICE V: Care of Clients with Physiologic and Psychosocial Alterations, Part C
November 17 & 18, 2018

Situation - You are a staff nurse in a psychiatric unit, use of therapeutic communication is one of your
nursing responsibilities.

96. Verbal communication is the use of words when talking to your patient.
A. understandable phase
B. content
C. the circumstances
D. understandable sentences

97. Context of a verbal communication is the:


A. use of understandable sentences
B. use of literal words
C. use of clear sentence
D. environmental where communication occurs

98. Non-verbal communication is the behavior that accompanies verbal communication. Which of the
following is NOT an indicator of this?
A. Eye Contact
B. Word representing an object
C. Grunts and groans
D. Bochy language

99. Which of the following give meaning and context to the message?
A. Process
B. Phrases and sentences
C. Context
D. Thoughts and feelings

100. Which of the following situation is an example of incongruent message?


A. When the nurse means what she says
B. When the words and behavior of the nurse agree
C. When what the nurse says and does do not agree
D. When content and process agree
NURSING PRACTICE V: Care of Clients with Physiologic and Psychosocial Alterations, Part C
November 17 & 18, 2018

KEY ANSWER FOR NURSING PRACTICE V

1.B 11.A 21.B 31.D 41.B 51.A 61.C 71.C 81.D 91.A
2.D 12.B 22.C 32.C 42.D 52.A 62.D 72.B 82.C 92.C
3.D 13.D 23.B 33.B 43.B 53.B 63.C 73.C 83.A 93.C
4.B 14.C 24.A 34.D 44.A 54.D 64.A 74.C 84.B 94.D
5.D 15.C 25.A 35.A 45.A 55.C 65.A 75.C 85.A 95.C
6.C 16.A 26.C 36.C 46.C 56.A 66.D 76.C 86.C 96.B
7.A 17.D 27.D 37.D 47.D 57.A 67.B 77.D 87.D 97.D
8.A 18.C 28.C 38.B 48.B 58.A 68.A 78.B 88.D 98.B
9.A 19.D 29.A 39.B 49.A 59.A 69.D 79.D 89.A 99.D
10.B 20.D 30.A 40.D 50.C 60.A 70.C 80.A 90.A 100.C

You might also like